Unit 3: Gynecology

Pataasin ang iyong marka sa homework at exams ngayon gamit ang Quizwiz!

A 67-year-old G3P3 woman presents with severe pelvic protrusion several years following an abdominal hysterectomy. She denies any incontinence. She failed conservative management with a pessary. As a result, she underwent a vaginal surgical repair where the pubocervical fascia was plicated in the midline, as well as laterally to the arcus tendineus fascia (white line). What defect was repaired in this patient? A. Cystocele B. Rectocele C. Uterine prolapse D. Enterocele E. Urethral diverticulum

A. Central and lateral cystoceles are repaired by fixing defects in the pubocervical fascia or reattaching it to the sidewall, if separated from the white line. Defects in the rectovaginal fascia are repaired in rectoceles. Uterine prolapse is surgically treated by a vaginal hysterectomy, but this patient already had a hysterectomy. Enteroceles are repaired by either vaginal or abdominal enterocele repairs. Vaginal vault prolapse is treated either by supporting the vaginal cuff to the uterosacral or sacrospinous ligaments, or by sacrocolpopexy. Urethral diverticulum does not present with severe pelvic protrusion.

A 22-year-old G0 woman presents with worsening pelvic pain. She previously underwent a laparoscopic ablation of endometriosis followed by continuous oral contraceptive pills. She had short-term relief from this approach, but now has failed this treatment and is seeking additional medical management. Which of the following mechanisms best explains how a gonadotropin releasing hormone (GnRH) agonist would help alleviate her symptoms? A. Down-regulation of the hypothalamic-pituitary gland production B. Up-regulation of the hypothalamic-pituitary gland production C. Suppression of both LH and FSH mid-cycle surges D. Induction of a pseudopregnancy state E. Competitive inhibitor for estrogen receptors

A. Gonadotropin-releasing hormone (GnRH) agonists are analogues of naturally occurring gonadotropin-releasing hormones that down-regulate hypothalamic-pituitary gland production and the release of luteinizing hormone and follicle-stimulating hormone leading to dramatic reductions in estradiol level. Numerous clinical trials show GnRH agonists are more effective than placebo and as effective as Danazol in relieving endometriosis-associated pelvic pain. Danazol, a 17-alpha-ethinyl testosterone derivative, suppresses the mid-cycle surges of LH and FSH. Combined estrogen and progestin therapy in oral contraceptives produces the pseudopregnancy state.

A 25-year-old G1P1 woman who is breastfeeding her 2 ½-week-old comes to the office with left breast pain and fever. The symptoms began earlier today and are not relieved by acetaminophen. She has no known drug allergies. Her vital signs are: blood pressure 120/60; pulse 64; temperature 99.9° F (37.7° C). On exam, there is erythema on the upper outer quadrant of the left breast, which is tender to touch; there are no palpable masses. What is the most appropriate antibiotic therapy for this patient? A. Dicloxacillin B. Erythromycin C. Doxycycline D. Gentamicin E. Cefotetan

A. Most postpartum mastitis is caused by staphylococcus aureus, so a penicillin-type drug is the first line of treatment. Dicloxacillin is used due to the large prevalence of penicillin resistant staphylococci. Erythromycin may be used in penicillin allergic patients. Doxycycline, gentamicin, and cefotetan are not appropriate antibiotics for treatment of mastitis.

A 60-year-old G4P4 woman presents with a two-year history of urine leakage with activity such as coughing, sneezing and lifting. Her past medical history is significant for vaginal deliveries of infants over 9 pounds. She had a previous abdominal hysterectomy and bilateral salpingo-oophorectomy for uterine fibroids. She is on vaginal estrogen for atrophic vaginitis. Physical examination reveals no anterior, apical or posterior wall vaginal prolapse. The vagina is well-estrogenized. Post-void residual is normal. Q-tip test shows a straining angle of 60 degrees from the horizontal. Cough stress test shows leakage of urine synchronous with the cough. Cystometrogram reveals the absence of detrusor instability. The patient failed pelvic muscle exercises and is not interested in an incontinence pessary. Which of the following is the best surgical option for this patient? A. Retropubic urethropexy B. Needle suspension C. Anterior repair D. Urethral bulking procedure E. Colpocleisis

A. Genuine stress incontinence (GSI) is the loss of urine due to increased intra-abdominal pressure in the absence of a detrusor contraction. The majority of GSI is due to urethral hypermobility (straining Q-tip angle >30 degrees from horizon). Some (<10%) of GSI is due to intrinsic sphincteric deficiency (ISD) of the urethra. Patients can have both hypermobility and ISD. Retropubic urethropexy such as tension-free vaginal tape and other sling procedures have the best five-year success rates for patients with GSI due to hypermobility. Needle suspensions and anterior repairs have lower five-year success rates for GSI. Urethral bulking procedures are best for patients with ISD, but with little to no mobility of the urethra. Colpocleisis is one option to treat uterine prolapse, and is not indicated for urinary incontinence.

A 62-year-old G5P5 woman presents with a seven-month history of pelvic pain and pressure, as well as abdominal distention and bloating. She experiences occasional constipation, but no melena or hematochezia. She also has mild to moderate urinary frequency without dysuria, hematuria or flank pain. Her medical history is significant for hypertension and obesity. She went through menopause 12 years ago and has never been on hormone therapy. She reports one episode of light vaginal bleeding several months ago. Her family history is significant for postmenopausal ovarian cancer in her mother and maternal aunt, but is otherwise negative for breast, endometrial or colon cancer. Pelvic examination is remarkable for vaginal atrophy, cervical stenosis and difficult uterine and adnexal assessment due to her body habitus. What is the most appropriate next step in the management of this patient? A. Transvaginal ultrasound B. CT scan of the abdomen and pelvis C. Colonoscopy D. Hysteroscopy E. Diagnostic laparoscopy

A. Given the patient's age, nonspecific abdomino-pelvic symptoms, recent postmenopausal bleeding episode and family history of ovarian cancer, a transvaginal ultrasound is the next best step as it is more sensitive than CT for evaluation of the uterus and adnexa. Colonoscopy is useful for colorectal cancer screening, as well as evaluation of the patient's gastrointestinal symptoms, but would not provide information regarding pelvic anatomy. Diagnostic laparoscopy would be a more invasive procedure that could be performed as indicated, after these other diagnostic studies. Hysteroscopy might be useful based on the ultrasound results, since it might be difficult to perform an endometrial biopsy in the office.

A 16-year-old girl is brought by her mother to her primary care physician requesting that her daughter have a gynecological examination. She is fearful that her daughter is sexually active. A private conversation with the girl reveals that she is sexually active and is on oral contraceptives prescribed by a community clinic. Having seen what her older sister went through when she became sexually active, she prefers not to tell her mother about her sexual activity for fear of severe punishment. The pelvic examination is normal. Which is the most appropriate action at this time? A. Tell the mother that her daughter is healthy B. Insist to the daughter that she disclose her sexual activity to her mother C. Reassure the mother that her daughter's sexual activity is a normal activity for most girls her age D. Speak to the mother openly about her daughter's condition E. Disclosure of any medical information is forbidden and a major HIPPA violation

A. It is not appropriate to discuss the daughter's sexuality with her mother, although her mother has the right to know about her daughter's overall health as her legal guardian. The daughter should be encouraged to have an open conversation with her mother, but that this is entirely up to her discretion and that she will be supported in whatever decision she ultimately makes about disclosing her sexual activity.

A 30-year-old G1P1 woman presents with a history of chronic vulvar pruritus. The itching is so severe that she scratches constantly and is unable to sleep at night. She reports no significant vaginal discharge or dyspareunia. She does not take antibiotics. Her medical history is unremarkable. Pelvic examination reveals normal external genitalia with marked lichenification (increased skin markings) and diffuse vulvar edema and erythema as shown in picture below. Saline microscopy is negative. Potassium hydroxide testing is negative. Vaginal pH is 4.0. The vaginal mucosa is normal. Which of the following is the most likely diagnosis in this patient? A. Lichen simplex chronicus B. Lichen sclerosus C. Lichen planus D. Candidiasis E. Vulvar cancer

A. Lichen simplex chronicus, a common vulvar non-neoplastic disorder, results from chronic scratching and rubbing, which damages the skin and leads to loss of its protective barrier. Over time, a perpetual itch-scratch-itch cycle develops, and the result is susceptibility to infection, ease of irritation and more itching. Symptoms consist of severe vulvar pruritus, which can be worse at night. Clinical findings include thick, lichenified, enlarged and rugose labia, with or without edema. The skin changes can be localized or generalized. Diagnosis is based on clinical history and findings, as well as vulvar biopsy. Treatment involves a short-course of high-potency topical corticosteroids and antihistamines to control pruritus.

A 20-year-old G2P2 healthy woman presents for her post-partum check six weeks after a full term normal spontaneous delivery. She has a 13 month old in addition to the six-week newborn, and is already feeling overwhelmed. She desires a reliable form of contraception. On exam, her vital signs are normal. BMI is 27. The remainder of the exam is unremarkable. Of the following, what is the most effective and appropriate form of contraception for this patient? A. Intrauterine device B. Tubal ligation C. Depo-Provera® D. Oral contraceptive pills E. Essure®

A. Long-acting reversible contraceptives (LARC) methods such as contraceptive implants and intrauterine devices are a good option for this patient. Despite high up-front costs and the need for office visits for insertion and removal, LARC methods provide many distinct advantages over other contraceptive methods as Depo-Provera® and oral contraceptives. While Depo-Provera is an effective form of contraception, it may not be the best choice in this woman with a high BMI. For this young mother who desires a reversible, but reliable form of contraception, the high effectiveness, continuation rate and user satisfaction of LARC methods would be of most benefit. Emerging evidence indicates that increasing the use of LARC methods also could reduce repeat pregnancy among adolescent mothers and repeat abortions among women seeking induced abortion. ("Increasing Use of Contraceptive Implants and Intrauterine Devices To Reduce Unintended Pregnancy," ACOG Committee Opinion, No. 450, 2009). Tubal ligation and Essure® are permanent and are not appropriate for this patient.

A 28-year-old G0 woman presents for preoperative counseling. She has a suspected diagnosis of severe endometriosis and has failed conservative medical management with OCPs. Her symptoms include severe pelvic pain especially prior to and during menstruation, and deep dyspareunia. On pelvic examination, she has uterosacral nodularity and tenderness throughout both adnexa. Ultrasound reveals normal ovaries bilaterally. Which of the following procedure is the best option for this patient? A. Laser ablation B. Ovarian cystectomy C. Salpingectomy D. Bilateral salpingo-oophorectomy E. Total laparoscopic hysterectomy and bilateral salpingo-oophorectomy

A. Surgery is the gold standard in the diagnosis of endometriosis, but often is not the initial treatment as suspected endometriosis is often managed medically. The role of surgery is often to manage the symptoms of endometriosis, often pelvic pain. As such, surgery may be conservative (laparoscopic ablation or excision of implants, excision of endometriomas) or definitive (total hysterectomy/BSO). In this young nulliparous patient, definitive surgery is not indicated, and only laser ablation would be recommended. As she has no evidence of adnexal masses or enlarged ovaries on ultrasound, it is unlikely that ovarian cystectomy would be needed. There is no indication for salpingectomy.

A 46-year-old female is scheduled to undergo a total laparoscopic hysterectomy for menorrhagia. She comes into your office to discuss the effects a hysterectomy will have on her sex life. She is concerned about how a hysterectomy will affect her "womanhood" and read in a woman's magazine that the cervix, as well as the uterus, is necessary for orgasm. Which of the following organs is most responsible for sexual arousal? A. Vulva B. Cervix C. Uterus D. Ovary E. All of the above

A. The external genitalia, specifically the clitoris, are essential organs involved in the arousal component of the sexual response cycle, which consists of desire, arousal, orgasm, and resolution. Hormones such as androgen and estrogen (produced by the ovaries) are key to desire, while genital mechanisms such as clitoral, labial, and vaginal engorgement are key to arousal. With arousal and adequate sensory stimulation, orgasm ultimately may occur consisting of repeated motor contractions of the pelvic floor including uterine and vaginal smooth muscle contractions.

A 26-year-old G2P2 woman presents with a new onset of vulvar burning and irritation. She is sexually active with a new male partner. She is using oral contraception for birth control and did not use a condom. She thought she had a cold about 10 days ago. Which of the following is the most likely diagnosis in this patient? A. Herpes simplex virus B. Primary syphilis C. Secondary syphilis D. Human immunodeficiency virus E. Trichomonas

A. The patient is most likely infected with herpes. Herpes simplex virus is a highly contagious DNA virus. Initial infection is characterized by viral-like symptoms preceding the appearance of vesicular genital lesions. A prodrome of burning or irritation may occur before the lesions appear. With primary infection, dysuria due to vulvar lesions can cause significant urinary retention requiring catheter drainage. Pain can be a very significant finding as well. Treatment is centered on care of the local lesions and the symptoms. Sitz baths, perineal care and topical Xylocaine jellies or creams may be helpful. Anti-viral medications, such as acyclovir, can decrease viral shedding and shorten the course of the outbreak somewhat. These medications can be administered topically or orally. Syphilis is a chronic infection caused by the Treponema pallidum bacterium. Transmission is usually by direct contact with an infectious lesion. Early syphilis includes the primary, secondary, and early latent stages during the first year after infection, while latent syphilis occurs after that and the patient usually has a normal physical exam with positive serology. In primary syphilis, a painless papule usually appears at the site of inoculation. This then ulcerates and forms the chancre, which is a classic sign of the disease. Left untreated, 25% of patients will develop the systemic symptoms of secondary syphilis, which include low-grade fever, malaise, headache, generalized lymphadenopathy, rash, anorexia, weight loss, and myalgias. This patient's symptoms are less consistent with syphilis, but she should still be tested for it. Human immunodeficiency virus is an RNA retrovirus transmitted via sexual contact or sharing intravenous needles. Vulvar burning, irritation or lesions are not typically noted with this disease, although generalized malaise can be. HIV can present with many different signs and symptoms, therefore risk factors should be considered, and testing offered. Trichomonas is a protozoan and is transmitted via sexual contact. It typically presents with a non-specific vaginal discharge. It does not have a systemic manifestation.

A 36-year-old G2P2 woman presents with irregular vaginal bleeding. Six weeks ago, she had her first Depo-Provera® injection and now she has unpredictable bleeding. She is concerned by these symptoms. She has a history of hypertension but is currently on no medications. Vital signs reveal: blood pressure 130/90; weight 188 pounds; height 5 feet 5 inches; BMI 31.4kg/m2. Which of the following is the most appropriate next step in the management of this patient? A. Reassurance B. Begin oral contraceptives C. Begin estrogen D. Insert etonogestrel implant (Implanon) E. Perform an endometrial biopsy

A. The patient should be reassured since initially after Depo-Provera injection there may be unpredictable bleeding. This usually resolves in 2-3 months. In general, after one year of using Depo-Provera, nearly 50% of users have amenorrhea.

A 26-year-old lesbian has chronic herpetic lesions on her lip. She is concerned about the affect this will have on her partner when she performs cunnilingus on her. Physical examination is normal except for a herpetic lesion on the lip, which has been diagnosed as Herpes type 1 in the past. Acyclovir is prescribed. Which of the following is the most appropriate advice to this patient? A. Use a dental dam when having oral sex B. Provide a prescription for acylovir for her partner to take before and after oral sex C. Apply a thin layer of petrolatum jelly over her lips while having oral sex D. Inform her that the Herpes virus type 1 is not contagious to the genitalia E. Discontinue oral sex

A. Use of a dental dam or a latex condom cut down the middle is effective in avoiding infection. Type 1 virus can cause ulcers on the vulva, as it is contagious. Prophylaxis of the partner with acyclovir is not a recommended strategy to prevent transmission, and petrolatum jelly is not an effective barrier.

A 48-year-old G0 comes to the office for a health maintenance examination. She is healthy and not taking any medications. She has no history of abnormal Pap tests or sexually transmitted infections. She is not currently sexually active. Her menstrual cycles are normal and her last cycle was three weeks ago. She smokes one pack of cigarettes per day. Her mother was diagnosed with endometriosis and had a hysterectomy and removal of the ovaries at age 38. She is 5 feet 4 inches tall and weighs 130 pounds. On pelvic examination, the patient has a palpable left adnexal mass. An ultrasound was obtained, which showed a 4 cm complex left ovarian cyst and a 2 cm simple cyst on the right ovary. What is the most appropriate next step in the management of this patient? A. Oral contraceptives B. Repeat ultrasound in two months C. CT scan of the abdomen and pelvis D. Needle aspiration of the cyst E. Abdominal hysterectomy and bilateral salpingo-oophorectomy (TAH/BSO)

B. A repeat ultrasound is the most appropriate next step, as this is most likely a hemorrhagic cyst which will resolve on its own. Oral contraceptives are contraindicated in this patient, as she is older than 35 and smokes. A CT scan of the pelvis will not add any more information. Needle aspiration is not the standard of care in this asymptomatic premenopausal patient. There is no indication to proceed with a TAH/BSO.

A 24-year-old G0 woman presents with a one-year history of introital and deep thrust dyspareunia. She has a two-year history of severe dysmenorrhea, despite the use of oral contraceptives. She also reports significant urinary frequency, urgency, and nocturia. A recent urine culture was negative. She underwent a diagnostic laparoscopy six months ago that showed minimal endometriosis with small implants in the posterior cul de sac only, which were ablated with a CO2 laser. What is the most likely diagnosis in this patient? A. Acute cystitis B. Interstitial cystitis C. Acute urethral syndrome D. Acute urethritis E. Salpingitis

B. Interstitial cystitis (IC) is a chronic inflammatory condition of the bladder, which is clinically characterized by recurrent irritative voiding symptoms of urgency and frequency, in the absence of objective evidence of another disease that could cause the symptoms. Pelvic pain is reported by up to 70% of women with IC and, occasionally, it is the presenting symptom or chief complaint. Women may also experience dyspareunia. The specific etiology is unknown, but IC may have an autoimmune and even hereditary component.

A 64-year-old G2P2 woman presents with a 12-month history of severe vulvar pruritus. She has applied multiple over-the-counter topical therapies without improvement. She has no significant vaginal discharge. She has severe dyspareunia at the introitus and has stopped having intercourse because of the pain. Her past medical history is significant for allergic rhinitis and hypertension. On pelvic examination the external genitalia show loss of the labia minora with resorption of the clitoris (phimosis). The vulvar skin appears thin and pale and involves the perianal area as in the picture below. No ulcerations are present. The vagina is mildly atrophic, but appears uninvolved. Which of the following is the most likely diagnosis in this patient? A. Squamous cell hyperplasia B. Lichen sclerosus C. Lichen planus D. Candidiasis E. Vulvar cancer

B. Lichen sclerosus is a chronic inflammatory skin condition that most commonly affects Caucasian premenarchal girls and postmenopausal women. The exact etiology is unknown, but is most likely multifactorial. Patients typically present with extreme vulvar pruritus and may also present with vulvar burning, pain and introital dyspareunia. Early skin changes include polygonal ivory papules involving the vulva and perianal areas, waxy sheen on the labia minora and clitoris, and hypopigmentation. The vagina is not involved. More advanced skin changes may include fissures and erosions due to a chronic itch-scratch-itch cycle, mucosal edema and surface vascular changes. Ultimately, scarring with loss of normal architecture, such as introital stenosis and resorption of the clitoris (phimosis) and labia minora, may occur. Treatment involves use of high-potency topical steroids. There is less than a 5% risk of developing squamous cell cancer within a field of lichen sclerosus.

A 20-year-old G0 college student presents with a one-month history of profuse vaginal discharge and mid-cycle vaginal spotting. She uses oral contraceptives and she thinks her irregular bleeding is due to the pill. She is sexually active and has had a new partner within the past three months. She reports no fevers or lower abdominal pain. She has otherwise been healthy. On pelvic examination, a thick yellow endocervical discharge is noted. Saline microscopy reveals multiple white blood cells, but no clue cells or trichomonads. Potassium hydroxide testing is negative. Vaginal pH is 4.0. No cervical motion tenderness or uterine/adnexal tenderness is present. Testing for gonorrhea and chlamydia is performed, but those results will not be available for several days and the student will be leaving for Europe tomorrow. Which of the following is the most appropriate treatment for this patient? A. Metronidazole and erythromycin B. Ceftriaxone and azithromycin C. Ampicillin and doxycycline D. Azithromycin and doxycycline E. No treatment is necessary until all tests results are known

B. Mucopurulent cervicitis (MPC) is characterized by a mucopurulent exudate visible in the endocervical canal or in an endocervical swab specimen. MPC is typically asymptomatic, but some women have an abnormal discharge or abnormal vaginal bleeding. MPC can be caused by Chlamydia trachomatis or Neisseria gonorrhoeae; however, in most cases neither organism can be isolated. Patients with MPC should be tested for both of these organisms. The results of sensitive tests for C. trachomatis or N. gonorrhoeae (e.g. culture or nucleic acid amplification tests) should determine the need for treatment, unless the likelihood of infection with either organism is high or the patient is unlikely to return for treatment. Antimicrobial therapy should include coverage for both organisms, such as azithromycin or doxycycline for chlamydia and a cephalosporin or quinolone for gonorrhea. Uncomplicated cervicitis, as in this patient, would require only 125 mg of Ceftriaxone in a single dose. Ceftriaxone 250 mg is necessary for the treatment of upper genital tract infection or pelvic inflammatory disease (PID).

A 45-year-old G2P2 woman underwent an abdominal hysterectomy for a large fibroid uterus via a low transverse skin incision. Her postoperative course was significant for new onset right lower quadrant pain and numbness, radiating into the right inguinal area and medial thigh. Her pain was exacerbated by adduction of her right thigh. On abdominal examination, there is a well-healed low transverse incision. Her pain is reproduced with adduction of the right thigh. There is decreased sensation to light touch and pinprick over the right inguinal area and right medial thigh. Patellar reflexes are 2+ and symmetric. Entrapment of which of the following nerves is the most likely cause of her pain? A. Obturator nerve B. Ilioinguinal nerve C. Lateral femoral cutaneous nerve D. Femoral nerve E. Iliohypogastric nerve

B. Nerve entrapment syndrome is a commonly misdiagnosed neuropathy that can complicate pelvic surgical procedures performed through a low transverse incision. The nerves at risk are the iliohypogastric nerve (T-12, L-1) and the ilioinguinal (T-12, L-1) nerve. These two nerves exit the spinal column at the 12th vertebral body and pass laterally through the psoas muscle before piercing the transversus abdominus muscle to the anterior abdominal wall. Once at the anterior superior iliac spine, the iliohypogastric nerve courses medially between the internal and external oblique muscles, becoming cutaneous 1 cm superior to the superficial inguinal ring. The iliohypogastric nerve provides cutaneous sensation to the groin and the skin overlying the pubis. The ilioinguinal nerve follows a similar, although slightly lower, course as the iliohypogastric nerve where it provides cutaneous sensation to the groin, symphysis, labium and upper inner thigh. These nerves may become susceptible to injury when a low transverse incision is extended beyond the lateral border of the rectus abdominus muscle, into the internal oblique muscle. Symptoms are attributed to suture incorporation of the nerve during fascial closure, direct nerve trauma with subsequent neuroma formation, or neural constriction due to normal scarring and healing. Damage to the obturator nerve, which can occur during lymph node dissection would result in the inability of the patient to adduct the thigh.

A 48-year-old G0 woman presents to the office for preoperative counseling. She has severe endometriosis that has failed medical management, and she is planning to undergo a robotic total hysterectomy and salpingo-oophorectomy. She is concerned about developing a "dropped bladder" following her surgery, since both her mother and aunt have undergone surgery for this condition. She reports no urinary incontinence or other urinary or bowel symptoms. She is in good health and exercises with running and weight lifting. Pelvic examination reveals a well-estrogenized vagina, a normal nulliparous cervix, anteverted uterus, and mildly tender adnexa without masses. Which of the following is likely to increase her risk of subsequent development of pelvic organ prolapse? A. Age B. Family History C. Endometriosis D. Exercising E. Hysterectomy

B. Risk factors for the development of pelvic organ prolapse are increasing parity, increasing age, obesity, some connective tissue disorders (Ehlers-Danlos syndrome), and chronic constipation. Vaginal delivery is associated with a higher risk of POP than Cesarean delivery. It is unclear whether occupations that require heavy lifting increase the risk of POP. Women with a family history of POP have up to a 2.5 fold increase in prolapse. Although hysterectomy is associated with an increased risk of apical prolapse, studies show mixed results on the role of hysterectomy in the development of prolapse. The risk of future prolapse may be highest when hysterectomy is performed in women with existing prolapse, while the risk in women with normal pelvic support is less clear.

A 54-year-old woman presents with a breast mass she noticed two months ago. She has no family history of breast cancer. On exam, there is a 2 cm mass palpable in the upper outer quadrant of the left breast. There are no other masses noted and no palpable lymphadenopathy. A fine needle aspiration returns bloody fluid and reduces the size of the mass to 1 cm. In addition to obtaining a mammogram, what is the most appropriate next step in the management of this patient? A. Repeat exam in two months B. Excisional biopsy of the mass C. Obtain a breast MRI D. Perform a lumpectomy and lymph node dissection E. Follow-up in one year if mammogram is normal

B. The first noticeable symptom of breast cancer is typically a lump that feels different from the rest of the breast tissue. More breast cancer cases are discovered when the woman feels a lump. Breast cancer can also present with a spontaneous bloody nipple discharge. Even though the mass decreased in size after aspiration, the bloody discharge obtained obligates an excisional biopsy be performed to rule out breast cancer. If clear discharge is obtained on aspiration and the mass resolves, reexamination in two months is appropriate to check that the cyst has not recurred. An MRI is not the appropriate next step and lumpectomy with lymph node dissection is not yet indicated in this case. A normal mammogram does not rule out breast cancer, especially in the presence of bloody discharge.

A 60-year-old G2P2 woman presents with complaints of urinary frequency and urge incontinence. Past medical history is unremarkable. She is on no medications. Pelvic examination reveals no evidence of pelvic relaxation. Post void residual is normal. Urine analysis is negative. A cystometrogram reveals uninhibited detrusor contractions upon filling. Which of the following is the best treatment for this patient? A. Amitriptyline B. Oxybutynin C. Topical (vaginal) estrogen D. Pseudoephedrine E. Kegel exercises

B. The patient has the diagnosis of detrusor instability. The parasympathetic system is involved in bladder emptying and acetylcholine is the transmitter that stimulates the bladder to contract through muscarinic receptors. Thus, anticholinergics are the mainstay of pharmacologic treatment. Oxybutynin is one example. Although the tricyclic antidepressant, amitriptyline, has anticholinergic properties, its side effects do not make it an ideal choice. Vaginal estrogen has been shown to help with urgency, but not urge incontinence. Pseudoephedrine has been shown to have alpha-adrenergic properties and may improve urethral tone in the treatment of stress incontinence. Kegel exercises or pelvic muscle training are used to strengthen the pelvic floor and decrease urethral hypermobility for the treatment of stress urinary incontinence.

A 57-year-old G2P2 is seen for a routine visit. She states she and her 75-year-old husband stopped having sexual intercourse three years ago when he had an operation for prostate cancer. Menopause occurred at age 50 and she denies taking hormones. Her husband now wishes to resume intercourse and is able to get an erection with sildenafil (Viagra). Attempts at intercourse have been unsuccessful due to the pain she experiences when insertion is attempted. Examination is normal except for a narrowed vagina with atrophic mucosa. Which of the following is the most appropriate recommendation at this time? A. Progesterone cream B. Estrogen cream C. Oral combined hormone therapy D. Petroleum jelly E. Vaginal dilators

B. The patient is not on hormone replacement therapy. With decreased estrogen production after menopause, the vaginal mucosa and other estrogen-dependent tissues can become atrophic. Topical estrogen therapy can restore the integrity of the vaginal epithelium, as well as the support tissues around the vagina. Long-term use may require addition of a progestin due to potential systemic absorption and effect on the endometrium. Dyspareunia (pain with intercourse) can often be improved with the use of estrogen cream used vaginally. Water based lubricants may be helpful, but petroleum jelly is contraindicated because it can cause irritation of the vaginal mucosa. This patient does not have any other menopausal symptoms that would be treated by oral combination HRT. Oral progesterone may help to decrease hot flushes, but progesterone cream would not alleviate vaginal atrophy.

A 23-year-old nulliparous woman presents with a painful nodule in her axilla for three days. She is healthy and has no personal or family history of breast disease. On exam, no abnormalities are seen on inspection and no breast mass is palpated. In the axillary area, shaved skin is noted and an erythematous raised 1 cm lesion is palpated and is slightly tender to touch. What is the most likely diagnosis? A. Paget's disease B. Folliculitis C. Fibroadenoma D. Supernumerary nipple E. Duct obstruction

B. This is a typical presentation for folliculitis which can occur with shaving the axillary hair. Paget's disease is a malignant condition of the breast that has the appearance of eczema and does not typically present in the axillary area. Fibroadenomas are common and are usually firm, painless and freely movable. A supernumerary nipple is a congenital variation and is typically located in the nipple line and not tender. A clogged milk duct can be present in the axillary region, but it is typically present in a woman who is breastfeeding.

A 25-year-old G1P1 woman comes to the office due to left breast pain and fever. She is breastfeeding her 2 ½-week-old infant. The symptoms began earlier in the day and are not relieved by acetaminophen. Her vital signs are: blood pressure 120/60; pulse 64; temperature 99.9° F (37.7° C). On exam, she has erythema on the upper outer quadrant of the left breast which is tender to touch. There are no palpable masses. In addition to starting oral antibiotics, what is the most appropriate next step in the management of this patient? A. Discontinue breastfeeding B. Begin ibuprofen C. Obtain a breast ultrasound D. Use a topical antifungal E. Apply breast binders

B. This patient has puerperal mastitis which most often occurs during the second to fourth week after delivery and the most appropriate next step in her management is to use ibuprofen in addition to acetaminophen for pain relief. She should be encouraged to continue breastfeeding or expressing their milk during treatment. Mastitis is usually treated as an outpatient. Patients are treated with oral or IV antibiotics, depending on the severity of infection. Cold compresses may reduce inflammation but are not indicated in the management of mastitis. Breast binders may increase breast pain. A breast ultrasound is not indicated if there is no suspicion of a breast abscess.

A 38-year-old G0 woman comes to the office for a health maintenance examination. She is healthy and not taking any medications. She has no history of abnormal Pap tests or sexually transmitted infections. Her menstrual cycles are normal and her last cycle was three weeks ago. Her mother was diagnosed with endometriosis and had a hysterectomy and removal of the ovaries at age 38. She is 5 feet 4 inches tall and weighs 130 pounds. On pelvic examination, the patient has a palpable left adnexal mass. An ultrasound was obtained, which showed a 4 cm complex left ovarian cyst and a 2 cm simple cyst on the right ovary. What is the most likely diagnosis in this patient? A. Endometrioma B. Hemorrhagic cyst C. Ovarian carcinoma D. Mature teratoma E. Polycystic ovaries

B. This patient most likely has a hemorrhagic cyst, considering her history and where she is in her menstrual cycle. Her mother's history of endometriosis does increase her risk; however, it is unlikely since she has never had any symptoms herself. Ovarian carcinoma would need to be ruled out, but it is unlikely in an otherwise asymptomatic premenopausal patient. A mature teratoma would have more pathognomonic findings on ultrasound. This patient does not have typical symptoms, body habitus or ultrasound findings for patients with polycystic ovaries.

A 42-year-old G2P2 woman presents with a two-week history of a thick, curdish white vaginal discharge and pruritus. She has not tried any over-the-counter medications. She is currently single and not sexually active. Her medical history is remarkable for recent antibiotic use for bronchitis. On pelvic examination, the external genitalia show marked erythema with satellite lesions. The vagina appears erythematous and edematous with a thick white discharge. The cervix appears normal and the remainder of the exam is unremarkable except for mild vaginal wall tenderness. Vaginal pH is 4.0. Saline wet prep reveals multiple white blood cells, but no clue cells or trichomonads. Potassium hydroxide prep shows the organisms. Which of the following is the most appropriate treatment for this patient? A. Clindamycin B. Azole cream C. Metronidazole D. Doxycycline E. Ciprofloxacin

B. Vulvovaginal candidiasis (VVC) usually is caused by C. albicans, but is occasionally caused by other Candida species or yeasts. Typical symptoms include pruritus and vaginal discharge. Other symptoms include vaginal soreness, vulvar burning, dyspareunia and external dysuria. None of these symptoms are specific for VVC. The diagnosis is suggested clinically by vulvovaginal pruritus and erythema with or without associated vaginal discharge. The diagnosis can be made in a woman who has signs and symptoms of vaginitis when either: a) a wet preparation (saline or 10% KOH) or Gram stain of vaginal discharge demonstrates yeasts or pseudohyphae; or b) a vaginal culture or other test yields a positive result for a yeast species. Microscopy may be negative in up to fifty percent of confirmed cases. Treatment for uncomplicated VVC consists of short-course topical Azole formulations (1-3 days), which results in relief of symptoms and negative cultures in 80%-90% of patients who complete therapy.

A 29-year-old G0 woman presents due to the inability to conceive for the last 18 months. She has a known history of endometriosis, which was diagnosed by laparoscopy three years ago. She has pelvic pain, which is controlled with non-steroidal anti-inflammatory drugs. Her cycles are regular. She is otherwise in good health and has been married for five years. Her husband had a semen analysis, which was normal. She had a hysterosalpingogram, which showed patent tubes bilaterally. She is getting frustrated that she has not yet achieved pregnancy and asks to proceed with fertility treatments. What is the most appropriate next step in the management of this patient? A. Reassurance and return in six months B. Administer a GnRH agonist C. Ovarian stimulation with clomiphene citrate D. Intrauterine insemination E. Proceed with in vitro fertilization

C. A patient with a known history of endometriosis, who is unable to conceive and has an otherwise negative workup for infertility, benefits from ovarian stimulation with clomiphene citrate, with or without intrauterine insemination. Waiting another six months is not appropriate as she has been trying to conceive for 18 months unsuccessfully. A GnRH agonist is used to control pelvic pain in endometriosis patients unresponsive to other hormonal treatments. In vitro fertilization and adoption can be offered if other treatments fail.

A 32-year-old G0 woman with a last menstrual period three weeks ago, presents with a three-month history of a malodorous vaginal discharge. She reports no pruritus or irritation. She has been sexually active with a new partner for the last four months. Her past medical history is unremarkable. Pelvic examination reveals normal external genitalia without rash, ulcerations or lesions. Some discharge is noted on the perineum. The vagina reveals only a thin, gray homogeneous discharge. The vaginal pH is 5.0. A wet prep is shown in the image below. Which of the following is the most appropriate treatment for this patient? A. Ceftriaxone B. Doxycycline C. Metronidazole D. Azithromycin E. Penicillin

C. Bacterial vaginosis is the most common cause of vaginitis. The infection arises from a shift in the vaginal flora from hydrogen peroxide-producing lactobacilli to non-hydrogen peroxide-producing lactobacilli, which allows proliferation of anaerobic bacteria. The majority of women are asymptomatic; however, patients may experience a thin, gray discharge with a characteristic fishy odor that is often worse following menses and intercourse. Modified Amsel criteria for diagnosis include three out of four of the following: 1) thin, gray homogenous vaginal discharge; 2) positive whiff test (addition of potassium hydroxide releases characteristic amine odor); 3) presence of clue cells on saline microscopy; and 4) elevated vaginal pH >4.5. Treatment consists of Metronidazole 500 mg orally BID for seven days, or vaginal Metronidazole 0.75% gel QHS for five days.

A 90-year-old G7P7 woman presents with severe vaginal prolapse. The entire apex, anterior and posterior wall are prolapsed beyond the introitus. She cannot urinate without reduction of the prolapse. Hydronephrosis is noted on ultrasound of the kidneys and thought to be related to the prolapse. She has a long-standing history of diabetes and cardiac disease. She has failed a trial of pessaries. Which of the following is the next best step in the management of this patient? A. Do nothing and observe B. Anterior and posterior repair C. Colpocleisis D. Sacrospinous fixation E. Sacrocolpopexy

C. Because of the hydronephrosis due to obstruction, intervention is required. Colpocleisis is a procedure where the vagina is surgically obliterated and can be performed quickly without the need for general anesthesia. Anterior and posterior repairs provide no apical support of the vagina. She will be at high risk of recurrent prolapse. The sacrospinous fixation (cuff to sacrospinous-coccygeus complex) or sacrocolpopexy (cuff to sacral promontory using interposed mesh) require regional or general anesthesia and is not the best option for this patient with high surgical morbidity.

A 17-year-old G0 female presents with a three-year history of severe dysmenorrhea shortly after menarche at age 14. Her menstrual cycles are regular with heavy flow. She has been treated with ibuprofen and oral contraceptives for the last year without significant improvement. She misses 2-3 days of school each month due to her menses. She has never been sexually active. Physical examination is remarkable for Tanner Stage IV breasts and pubic hair. Pelvic examination is normal, as is a pelvic ultrasound. Both the patient and her mother are concerned. What is the next best step in the management of this patient? A. Sonohysterogram B. CT scan of the pelvis C. Diagnostic laparoscopy D. MRI of the pelvis E. Hysterosalpingogram

C. Chronic pelvic pain is the indication for at least 40% of all gynecologic laparoscopies. Endometriosis and adhesions account for more than 90% of the diagnoses in women with discernible laparoscopic abnormalities, and laparoscopy is indicated in women thought to have either of these conditions. Often, adolescents are excluded from laparoscopic evaluation on the basis of their age, but several series show that endometriosis is as common in adolescents with chronic pelvic pain as in the general population. Therefore, laparoscopic evaluation of chronic pelvic pain in adolescents should not be deferred based on age. Laparoscopy can be both diagnostic and therapeutic in this patient in whom you suspect endometriosis. None of the other imaging modalities listed will help in the further workup of this patient.

A 76-year-old G3P3 woman presents to your office with worsening urinary incontinence for the past three months. She reports increased urinary frequency, urgency and nocturia. On examination, she has a mild cystocele and rectocele. A urine culture is negative. A post-void residual is 400 cc. Which of the following is the most likely diagnosis in this patient? A. Genuine stress incontinence B. Detrusor instability C. Overflow incontinence D. Functional incontinence E. Mixed incontinence

C. Overflow incontinence is characterized by failure to empty the bladder adequately. This is due to an underactive detrusor muscle (neurologic disorders, diabetes or multiple sclerosis) or obstruction (postoperative or severe prolapse). A normal post-void residual (PVR) is 50-60 cc. An elevated PVR, usually >300 cc, is found in overflow incontinence. Stress incontinence occurs when the bladder pressure is greater than the intraurethral pressure. Overactive detrusor contractions can override the urethral pressure resulting in urine leakage. The mixed variety includes symptoms related to stress incontinence and urge incontinence.

A 48-year-old G4P4 woman with last menstrual period four weeks ago presents with a one-year history of non-cyclical pelvic pain, dysmenorrhea and dyspareunia. She has a past history of endometriosis, diagnosed 10 years ago by laparoscopy. She had previously been on oral contraceptives for birth control and menstrual cycle regulation, but elected for permanent laparoscopic sterilization 14 months ago. Minimal endometriosis was noted at the time of laparoscopy. She now has recurrent symptoms and desires definitive treatment. Which of the following is the most appropriate surgical option for this patient?A 48-year-old G4P4 woman with last menstrual period four weeks ago presents with a one-year history of non-cyclical pelvic pain, dysmenorrhea and dyspareunia. She has a past history of endometriosis, diagnosed 10 years ago by laparoscopy. She had previously been on oral contraceptives for birth control and menstrual cycle regulation, but elected for permanent laparoscopic sterilization 14 months ago. Minimal endometriosis was noted at the time of laparoscopy. She now has recurrent symptoms and desires definitive treatment. Which of the following is the most appropriate surgical option for this patient? A. Hysteroscopy and dilation and curettage B. Diagnostic laparoscopy C. Hysterectomy with bilateral salpingo-oophorectomy D. Endometrial ablation E. Wedge resection of the ovaries

C. It is estimated that chronic pelvic pain is the principal preoperative indication for 10-12% of hysterectomies. Since the patient had a tubal ligation and does not desire any more children, the best option is removal of ovaries with or without a hysterectomy. Repeat laparoscopy with treatment of endometriosis and adhesions can be helpful; however, the patient will continue to be at increased risk of recurrent disease. An endometrial ablation or wedge resection of ovaries alone would not be very helpful in the setting of non-cyclical pain.

A 52-year-old G0 woman presents with long-standing vulvar and vaginal pain and burning. She has been unable to tolerate intercourse with her husband because of pain at the introitus. She has difficulty sitting for prolonged periods of time or wearing restrictive clothing because of worsening vulvar pain. She recently noticed that her gums bleed more frequently. She avoids any topical over-the-counter therapies because they intensify her pain. Her physical examination is remarkable for inflamed gingiva and a whitish reticular skin change on her buccal mucosa. A fine papular rash is present around her wrists bilaterally. Pelvic examination reveals white plaques with intervening red erosions on the labia minora as shown in below picture. A speculum cannot be inserted into her vagina because of extensive adhesions. The cervix cannot be visualized. Which of the following is the most likely diagnosis in this patient? A. Squamous cell hyperplasia B. Lichen sclerosus C. Lichen planus D. Genital psoriasis E. Vulvar cancer

C. Lichen planus is a chronic dermatologic disorder involving the hair-bearing skin and scalp, nails, oral mucous membranes and vulva. This disease manifests as inflammatory mucocutaneous eruptions characterized by remissions and flares. The exact etiology is unknown, but is thought to be multifactorial. Vulvar symptoms include irritation, burning, pruritus, contact bleeding, pain and dyspareunia. Clinical findings vary with a lacy, reticulated pattern of the labia and perineum, with or without scarring and erosions as well. With progressive adhesion formation and loss of normal architecture, the vagina can become obliterated. Patients may also experience oral lesions, alopecia and extragenital rashes. Treatment is challenging, since no single agent is universally effective and consists of multiple supportive therapies and topical high potency corticosteroids.

A 36-year-old G2P0 woman at 11 weeks gestational age requests a surgical termination of pregnancy. She had a manual vacuum aspiration last year and would like to undergo the same procedure again. She has chronic hypertension and diabetes well controlled on medications. Vital signs reveal a blood pressure of 120/80 and fasting blood glucose of 100. Which of the following is a contraindication for manual vacuum aspiration of this patient? A. Age B. Parity C. Gestational age D. Chronic hypertension E. Diabetes

C. Manual vacuum aspiration is more than 99% effective in early pregnancy (less than eight weeks). Age, parity and medical illnesses are not contraindications for manual vacuum aspiration. Although the risk of Asherman's syndrome increases with each subsequent pregnancy termination, this patient may still undergo surgical termination as long as she understands risks and benefits. Complications of pregnancy termination increase with increasing gestational age.

A 26-year-old G0 woman returns for a follow-up visit regarding endometriosis. She has been using NSAIDs to manage her pelvic pain, but had to miss four days of work in the last two months. She is sexually active with her husband of two years, although it has been more painful recently. She has regular menstrual cycles and is using condoms for contraception. On pelvic examination, she has localized tenderness in the cul de sac and there are no palpable masses. What is the most appropriate next step in the management of this patient? A. GnRH agonist B. Danazol C. Oral contraceptives D. Laparoscopy and ablation of endometriosis E. Progesterone intrauterine device

C. Oral contraceptives will be the next best choice for this patient. They provide negative feedback to the pituitary-hypothalamic axis which stops stimulation of the ovary resulting in ovarian suppression of sex hormone production, such as estrogen. Since estrogen stimulates endometrial tissue located outside of the endometrium and uterus, endometriosis can be suppressed by OCPs especially when prescribed in a continuous fashion (omit the week of placebo pills resulting in no withdrawal bleed). GnRH agonists also exert negative feedback, but can be used short term only and have more side effects. Danazol is a synthetic androgen used to treat endometriosis, but due to its androgenic side effects (weight gain, increased body hair and acne, and adverse affect on blood lipid levels) it is not usually the first choice of treatment. Laparoscopy is indicated in the patient who fails medical treatment and/or is planning pregnancy in the near future. A progesterone intrauterine device might potentially help alleviate some of her symptoms, but is not the best management for endometriosis.

A 65-year-old G3P3 woman presents with symptoms of vaginal pressure and heaviness, which seem to worsen towards the end of the day. She has a history of three vaginal deliveries. Her surgical history is significant for hysterectomy for abnormal vaginal bleeding at age 45. On exam, she is found to have a large pelvic prolapse. Which of the following is the most appropriate initial treatment of this patient's prolapse? A. Sacrospinous ligament suspension B. Transvaginal tape C. Pessary fitting D. Anterior repair E. Topical vaginal estrogen

C. Pessary fitting is the least invasive intervention for this patient's symptomatic prolapse. Although a sacrospinous ligament suspension would be an appropriate procedure for this patient, it is invasive and not an appropriate first step. Transvaginal tape is used for urinary incontinence and has no role in the management of this patient. An anterior repair can potentially help with her symptoms, depending on what is contributing most to her prolapse but, again, it is invasive. Topical estrogen is unlikely to properly treat her prolapse and related symptoms.

A 12-year-old girl is brought to the office by her mother who complains that her daughter has never been interested in dolls and pretty dresses, but prefers to play with tools and mechanical things. The mother also divulges that her brother is gay and is worried that her daughter will grow up as a lesbian and be stigmatized. A private conversation with the girl reveals that she is starting to show an interest in boys, and even has a "boyfriend." Examination reveals a normal pre-pubertal phenotype. Which of the following is the most appropriate course of action at this time? A. Encourage mother to support more gender-appropriate activities including dressing like and playing with other girls B. Refer the mother to a family counselor C. Reassure the mother that her daughter's behavior is normal D. Inform mother that her daughter may ultimately develop an alternative sexual lifestyle E. Have the patient return for a repeat assessment once she has begun menarche

C. The division of interests based on gender is inappropriate. Children should be encouraged to follow their own interests and desires. The girl has a normal physical exam; therefore, diagnostic studies are not indicated.

A 74-year-old G0 woman complains of vulvar pain. She reports that the pain is present every day and she has had it for the past year. It now limits her ability to exercise, and she is no longer able to have sexual relations with her partner. On exam, her BMI is 32; blood pressure is 100/60; and heart rate is 77. Her vulva has an ulcerated lesion near the left labial edge. Which of the following is the next best step in the management of this patient? A. Estrogen cream B. Clobetasol cream C. Vulva biopsy D. Laser vaporization of the lesion E. Vulvectomy

C. This patient has a vulvar lesion causing her pain. The next step is to perform a biopsy to evaluate for vulvar cancer. Estrogen cream and clobetasol (a high potency steroid) are treatments for vulvadynia. To diagnosis vulvadynia, all other causes of pain must first be excluded, including infectious etiologies as well as other vulvar conditions. Laser vaporization and vulvectomy are contraindicated until a definitive diagnosis is made.

A 23-year-old G0 woman comes to the office to discuss contraception. Her past medical history is remarkable for hypothyroidism and mild hypertension. She has a history of slightly irregular menses. Her best friend recently got a "patch," so she is interested in using a transdermal system (patch). Her vital signs are: blood pressure 130/84; weight 210 pounds; height 5 feet 4 inches. What is the most compelling reason for her to use a different method of contraception? A. Age B. Hypothyroidism C. Weight D. Unpredictable periods E. Her blood pressure

C. The patch has comparable efficiency to the pill in comparative clinical trials, although it has more consistent use. It has a significantly higher failure rate when used in women who weigh more than 198 pounds. The patch is a transdermal system that is placed on a woman's upper arm or torso (except breasts). The patch (Ortho Evra®) slowly releases ethinyl estradiol and norelgestromin, which establishes steady serum levels for seven days. A woman should apply one patch in a different area each week for three weeks, then have a patch-free week, during which time she will have a withdrawal bleed.

A 29-year-old G0 woman presents due to the inability to conceive for the last year. Her cycles are regular every 28 days, but she has very painful periods, occasionally requiring that she miss work despite the use of non-steroidal anti-inflammatory drugs (NSAIDs). She also reports painful intercourse, which is becoming a problem as she now tries to avoid intercourse, even though she would like to conceive. She is otherwise in good health and has been married for five years. She is 5 feet 4 inches tall and weighs 130 pounds. She has a history of pelvic inflammatory disease at age 19, for which she was hospitalized. Her mother had a history of ovarian cancer at age 49. On physical examination, she has abdominal and pelvic lower quadrant tenderness. Uterus is normal in size, but there is a slightly tender palpable left adnexal mass. A pelvic ultrasound shows a 5 cm left complex ovarian cyst and two simple cysts measuring 2 cm in the right ovary. What best explains the underlying pathophysiology of the disease process in this patient? A. Chronic pelvic inflammatory disease B. Family history of ovarian cancer C. Endometrial glands outside the uterine cavity D. Polycystic ovarian syndrome E. Functional hemorrhagic cysts

C. The patient has typical signs of endometriosis which is characterized by the presence of endometrial glands and stroma outside of the uterus. Endometriosis is present in about 30% of infertile woman. She does not have the signs and symptoms of chronic pelvic inflammatory disease. She also does not have the signs and symptoms of polycystic ovarian syndrome, which typically presents with oligomenorrhea in overweight patients. The complex ovarian cyst is most likely an endometrioma (chocolate cyst). The duration of her symptoms makes functional hemorrhagic cyst a less likely option.

A 33-year-old G3P3 woman presents to the office complaining of a new onset vaginal discharge of four days duration. The discharge is thick and white. She has noted painful intercourse and itching since the discharge began. Her vital signs are: blood pressure 120/76 and pulse 78. The pelvic examination reveals excoriations on the perineum, thick white discharge, and is otherwise non-contributory. What is the most likely diagnosis in this patient? A. Herpes simplex virus B. Primary syphilis C. Candida vaginalis D. Bacterial vaginosis E. Trichomonas

C. The patient is most likely has candida vaginalis. Clinically women have itching and thick white cottage cheese like discharge. They may also have burning with urination and pain during intercourse. Herpes simplex viral infections are characterized by viral like symptoms preceding the appearance of vesicular genital lesions. A prodrome of burning or irritation may occur before the lesions appear. With primary infection, dysuria due to vulvar lesions can cause significant urinary retention requiring catheter drainage. Pain can be a very significant finding as well. Syphilis is a chronic infection caused by the Treponema pallidum bacterium. Transmission is usually by direct contact with an infectious lesion. Early syphilis includes the primary, secondary, and early latent stages during the first year after infection, while latent syphilis occurs after that and the patient usually has a normal physical exam with positive serology. In primary syphilis, a painless papule usually appears at the site of inoculation. This then ulcerates and forms the chancre, which is a classic sign of the disease. Left untreated, 25% of patients will develop the systemic symptoms of secondary syphilis, which include low-grade fever, malaise, headache, generalized lymphadenopathy, rash, anorexia, weight loss, and myalgias. Bacterial vaginosis is due to an overgrowth of anaerobic bacteria and characterized by a grayish / opaque foul-smelling discharge. Trichomonas is a protozoan and is transmitted via sexual contact. It typically presents with a non-specific yellow or greenish vaginal discharge. It does not have a systemic manifestation.

A 42-year-old G2P2 woman presents with chronic pelvic pain of two years duration. She describes the pain as constant ever since she underwent a laparoscopic-assisted vaginal hysterectomy for menorrhagia and dysmenorrhea. She did not have any evidence of endometriosis or obvious ovarian pathology at the time of surgery. During the postoperative period, she developed pelvic pain and fever, and was diagnosed with a pelvic/vaginal cuff abscess that was treated with antibiotics and percutaneous drainage. Her pain persisted in the subsequent months. Follow-up imaging over the next two years indicated transient ovarian cysts. Her abdominal examination is notable for mild-moderate tenderness across the lower quadrants, and her pelvic examination is notable for severe tenderness at the vaginal cuff with fullness noted in the midline. Which of the following is the most likely diagnosis in this patient? A. Endometriosis B. Pelvic inflammatory disease C. Pelvic adhesive disease D. Ovarian remnant syndrome E. Ovarian cancer

C. The patient most likely has pelvic adhesive disease as a result of her prior hysterectomy. The development of a postoperative pelvic infection likely has contributed to the further development of pelvic adhesions involving the tubes and ovaries that were retained. Although she did have a pelvic infection, it is unlikely that her pain resulted from classic PID since her adnexa appeared normal at the time of the hysterectomy. It is likely that her tubes and ovaries were affected by the postoperative infection, and as a result she may develop chronic pain from the adhesive disease and tubal damage from that acute infection. The cyclical nature of the ovarian cyst essentially rules out ovarian cancer. Ovarian remnant syndrome occurs following surgical removal of the ovaries, with subsequent development of cyclical pain due to ovarian tissue that was left behind inadvertently.

A 29-year-old G3P0 woman presents for evaluation and treatment of pregnancy loss. Her past medical history is remarkable for three early (< 10 weeks gestation) pregnancy losses and a deep vein thrombosis two years ago. Her work up includes: prolonged dilute Russell viper venom test, elevated anticardiolipin antibodies, normal thyroid function, normal prolactin, and normal MRI of the pelvis. She wishes to get pregnant soon. In addition to aspirin, which of the following treatments is appropriate for this patient? A. No additional treatment B. Corticosteroid C. Heparin D. 17-OH progesterone E. Bromocriptine

C. The prolonged dilute Russell viper venom time, history of three early pregnancy losses, and a history of venous thrombosis leads one to suspect that the etiology of recurrent pregnancy loss is due to antiphospholipid antibody syndrome. The treatment is aspirin plus heparin. There is roughly a 75% success rate with combination therapy versus aspirin alone. There is conflicting evidence regarding steroid use for treatment. 17-OH progesterone is used for the prevention of preterm delivery and not recurrent pregnancy loss. Bromocriptine would be indicated for hyperprolactinemia.

A 32-year-old G0 woman presents with a one-month history of profuse vaginal discharge with mild odor. She has a new sexual partner with whom she has had unprotected intercourse. She reports mild to moderate irritation, pruritus and pain. She thought she had a yeast infection, but had no improvement after using an over-the-counter antifungal cream. She is concerned about sexually transmitted infections. Her medical history is significant for lupus and chronic steroid use. Pelvic examination shows normal external genitalia, an erythematous vagina with a copious, frothy yellow discharge and multiple petechiae on the cervix. Vaginal pH is 7. Which of the following findings on a wet prep explains the etiology of this condition? A. Hyphae B. Clue cells C. Trichomonads D. Lactobacilli E. Normal epithelial cells

C. This patient has signs and symptoms of trichomoniasis, which is caused by the protozoan, T. vaginalis. Many infected women have symptoms characterized by a diffuse, malodorous, yellow-green discharge with vulvar irritation. However, some women have minimal or no symptoms. Diagnosis of vaginal trichomoniasis is performed by saline microscopy of vaginal secretions, but this method has a sensitivity of only 60% to 70%. The CDC recommended treatment is metronidazole 2 grams orally in a single dose. An alternate regimen is metronidazole 500mg orally twice daily for seven days. The patient's sexual partner also should undergo treatment prior to resuming sexual relations.

A 27-year-old G0 woman presents with a one-year history of dysmenorrhea and dyspareunia. Pain, when present, is 7/10 in strength and requires that she miss work. She now avoids intercourse and no longer finds it pleasurable. She is otherwise in good health. Her last menstrual period was 17 days ago and her menses are typically 28 days apart. She had chlamydia once, at age 19. Physical examination is notable for mild tenderness on abdominal examination in the lower quadrants, and bilateral adnexal tenderness on pelvic examination. Uterus is normal in size and there is uterosacral ligament nodularity. What is the most likely diagnosis in this patient? A. Adenomyosis B. Chronic pelvic inflammatory disease C. Endometriosis D. Endometritis E. Leiomyoma

C. This patient has typical symptoms of endometriosis, including dysmenorrhea and dyspareunia. In addition, the nodularity along the back of the uterus along the uterosacral ligaments is suggestive of endometriosis. Chronic pelvic inflammatory disease would not present this far out from a known infection. Adenomyosis is endometrial glands embedded in the wall of the uterus. Endometritis is an infection of the endometrium. Premenstrual dysphoric disorder (PMDD) is a condition in which a woman has severe depressive symptoms, irritability, and tension before menstruation.

A 57-year-old G2P2 woman presents with a six-month history of urinary incontinence, urgency, and nocturia. She describes the amount of urine loss as large and lasting for several seconds. The urine loss occurs when she is standing or sitting and is not associated with any specific activity. A post-void residual is 50cc. What is the most likely cause of this patient's symptoms? A. Stress incontinence B. Overflow incontinence C. Urge incontinence D. Mixed incontinence E. Vesicovaginal fistula

C. This patient has urge incontinence, which is caused by overactivity of the detrusor muscle resulting in uninhibited contractions, which cause an increase in the bladder pressure over urethral pressure resulting in urine leakage. Stress incontinence is caused by an increase in intra-abdominal pressure (coughing, sneezing) when the patient is in the upright position. This increase in pressure is transmitted to the bladder that then rises above the intra-urethral pressure causing urine loss. Associated structural defects are cystocele or urethrocele. Overflow incontinence is associated with symptoms of pressure, fullness, and frequency, and is usually a small amount of continuous leaking. It is not associated with any positional changes or associated events. Mixed incontinence occurs when increased intra-abdominal pressure causes the urethral-vesical junction to descend causing the detrusor muscle to contract. A vesicovaginal fistula typically results in continuous loss of urine.

A 37-year-old G0 woman presents with a one-week history of a mildly painful vulvar ulcer. She reports no fevers, malaise or other systemic symptoms. She recently started use of a topical steroid ointment for a vulvar contact dermatitis. She is married and has no prior history of sexually transmitted infections. She reports no travel outside the United States by her husband or herself. Her last Pap smear, six months ago, was normal. A vulvar herpes culture later returns positive for herpes simplex virus type 2. A Rapid Plasma Reagin (RPR) is nonreactive, and HIV testing is negative. Which of the following is the most likely diagnosis in this patient? A. Primary HSV episode B. Recurrent HSV-1 episode C. Recurrent HSV-2 episode D. Atypical HSV episode E. Contact dermatitis

C. Two serotypes of HSV have been identified: HSV-1 and HSV-2. Most cases of recurrent genital herpes are caused by HSV-2. Up to 30% of first-episode cases of genital herpes are caused by HSV-1, but recurrences are much less frequent for genital HSV-1 infection than genital HSV-2 infection. Genital HSV infections are classified as initial primary, initial nonprimary, recurrent and asymptomatic. Initial, or first-episode primary genital herpes is a true primary infection (i.e. no history of previous genital herpetic lesions, and seronegative for HSV antibodies). Systemic symptoms of a primary infection include fever, headache, malaise and myalgias, and usually precede the onset of genital lesions. Vulvar lesions begin as tender grouped vesicles that progress into exquisitely tender, superficial, small ulcerations on an erythematous base. Initial, nonprimary genital herpes is the first recognized episode of genital herpes in individuals who are seropositive for HSV antibodies. Prior HSV-1 infection confers partial immunity to HSV-2 infection and thereby lessens the severity of type 2 infection. The severity and duration of symptoms are intermediate between primary and recurrent disease, with individuals experiencing less pain, fewer lesions, more rapid resolution of clinical lesions and shorter duration of viral shedding. Systemic symptoms are rare. Recurrent episodes involve reactivation of latent genital infection, most commonly with HSV-2, and are marked by episodic prodromal symptoms and outbreaks of lesions at varying intervals and of variable severity. Clinical diagnosis of genital herpes should be confirmed by viral culture, antigen detection or serologic tests. Treatment consists of antiviral therapy with acyclovir, famciclovir or valacyclovir.

A 27-year-old G0 woman presents with a three-year history of dyspareunia. She reports a history of always having painful intercourse, but she is now unable to tolerate intercourse at all. She has avoided sex for the last six months. She describes severe pain with penile insertion. On further questioning, she reports an inability to use tampons because of painful insertion. She also notes a remote history of frequent yeast infections while she was on antibiotics for recurrent sinusitis that occurred years ago. Her medical history is unremarkable, and she is not on medications. Pelvic examination is remarkable for normal appearing external genitalia. Palpation of the vestibule with a Q-tip elicits marked tenderness and slight erythema. A normal-appearing discharge is noted. Saline wet prep shows only a few white blood cells, and potassium hydroxide testing is negative. Vaginal pH is 4.0. The cervix and uterus are unremarkable. Which of the following is the most likely diagnosis in this patient? A. Vaginal cancer B. Genital herpes infection C. Vestibulodynia D. Contact dermatitis E. Chlamydia infection

C. Vestibulodynia (formally vulvar vestibulitis) syndrome consists of a constellation of symptoms and findings limited to the vulvar vestibule, which include severe pain on vestibular touch or attempted vaginal entry, tenderness to pressure and erythema of various degrees. Symptoms often have an abrupt onset and are described as a sharp, burning and rawness sensation. Women may experience pain with tampon insertion, biking or wearing tight pants, and avoid intercourse because of marked introital dyspareunia. Vestibular findings include exquisite tenderness to light touch of variable intensity with or without focal or diffuse erythematous macules. Often, a primary or inciting event cannot be determined. Treatment includes use of tricyclic antidepressants to block sympathetic afferent pain loops, pelvic floor rehabilitation, biofeedback, and topical anesthetics. Surgery with vestibulectomy is reserved for patients who do not respond to standard therapies and are unable to tolerate intercourse.

A 42-year-old G3P3 woman comes to the office after noticing a breast mass while performing a breast self-exam. She is in good health and has normal menstrual cycles. Physical exam is significant for a 2 cm dominant breast mass. The remainder of the exam is normal. A mammogram obtained today shows no abnormalities. A fine needle aspiration was negative, and the mass persisted. What is the most appropriate next step in the management of this patient? A. Reassurance and observation B. Obtain an MRI of the chest C. Breast ultrasound D. Perform an excisional biopsy E. Repeat mammogram in two months

D. A specimen obtained on fine-needle aspiration (FNA) is examined both histologically and cytologically. An excisional biopsy should be performed when the results are negative, due to the possibility of a false-negative result. FNA can, however, prevent the need for other diagnostic testing and is the appropriate first step in the evaluation of a palpable breast mass. Breast ultrasound can be used to distinguish between a cyst and a solid mass. Fine needle aspiration under ultrasound guidance can help distinguish a fibroadenoma from a cyst and exclude cancer in certain situations. A normal mammogram does not rule out breast cancer and there is no need to repeat it in two months. There are no indications for obtaining an MRI of the chest in the initial diagnosis of this patient.

A 68-year-old G3P3 woman comes to the office due to breast tenderness. She is in good health and not taking any medications. Family history is significant for her 70-year-old sister recently diagnosed with breast cancer. On breast examination, her breasts have no lesions; there are no palpable masses, nodules or lymphadenopathy. Her last mammogram was four months ago and was normal. What is the most appropriate next step in the management of this patient? A. Order a mammogram B. Order a breast ultrasound C. Obtain genetic testing (BRCA-1 and BRCA-2 mutations) D. Reassurance E. Order a breast MRI

D. Age and gender are the greatest risk factors for developing breast cancer. Having one first-degree relative with breast cancer does increase the risk. A women's risk of developing breast cancer before menopause is increased if she is BRCA-1 or BRCA-2 positive; however, these genetic mutations occur in a low percentage of the general population. There is no indication for a mammogram since the patient's last mammogram was normal four months ago. Ultrasound and MRI would not add valuable information especially in the setting of a normal mammogram and no masses on physical examination. Genetic testing is not indicated in this case as there is no strong family history and the sister with breast cancer was postmenopausal at time of diagnosis.

A 36-year-old G0 woman presents to the emergency department accompanied by her female partner. The patient notes severe abdominal pain. She states that this pain began 2-3 days ago and was associated with diarrhea as well as some nausea. It has gotten progressively worse and she has now developed a fever. Neither her partner, nor other close contacts, report any type of viral illness. She had her appendix removed as a teenager. On examination, her temperature is 102.0°F (38.9°C), her abdomen is tender with mild guarding and rebound, and she has an elevated white count. On pelvic examination, she is exquisitely tender, such that you cannot complete the examination. Pelvic ultrasound demonstrates bilateral 3-4 cm complex masses. What is the most likely underlying pathogenesis of her illness? A. Diverticulitis B. Gastroenteritis C. Reactivation of an old infection D. Ascending infection E. Pyelonephritis

D. Although salpingitis is most often caused by sexually transmitted agents such as gonorrhea and chlamydia, any ascending infection from the genitourinary tract or gastrointestinal tract can be causative. The infection is polymicrobial consisting of aerobic and anaerobic organisms such as E. coli, Klebsiella, G. vaginalis, Prevotella, Group B streptococcus and/or enterococcus. Although diverticulitis and gastroenteritis should be part of the differential diagnosis initially, the specific findings on examination and ultrasound are more suggestive of bilateral tubo-ovarian abscesses. Even though this patient does not have the typical risk factors for salpingitis, the diagnosis should be considered and explained to the patient in a sensitive and respectful manner. The patient should also be questioned separate from her partner regarding the possibility of other sexual contacts.

A 16-year-old G0 female presents to the emergency department with a two-day history of abdominal pain, nausea and vomiting. She is sexually active with a new partner and is not using any form of contraception. On examination, her temperature is 100.2°F (37.9°C), and she has bilateral lower quadrant pain, with slight rebound and guarding. On pelvic examination, she has purulent cervical discharge and cervical motion tenderness. Her white count is 14,000/mcL. What is the most appropriate next step in the management of this patient? A. Oral amoxicillin clavunate and doxycycline B. Oral metronidazole and doxycycline C. IV metronidazole and doxycycline D. IV cefotetan and doxycycline E. No treatment until culture results are back

D. Although some patients can be treated with an outpatient regimen, this patient should be hospitalized for IV treatment, as she has nausea and vomiting so she might not be able to tolerate oral medications. While adolescents have no better outcomes from inpatient vs outpatient therapy, each patient should be assessed for compliance. It is important to treat aggressively in order to prevent the long-term sequelae of acute salpingitis. You would not wait for culture results before initiating treatment. Her recent sexual contacts should also be informed (by her and/or with her consent) and treated. According to the 2010 CDC treatment guidelines, there are two options for parenteral antibiotics covering both gonorrhea and chlamydia. Cefotetan or cefoxitin PLUS doxycycline or clindamycin PLUS gentamicin. For outpatient treatment, the 2010 CDC guidelines recommend ceftriaxone, cefoxitin, or other third-generation cephalosporin (such as ceftizoxime or cefotaxime) PLUS doxycycline WITH or WITHOUT metronidazole. There are alternative oral regimens as well. http://www.cdc.gov/std/treatment/2010/pid.htm

A 24-year-old G2P2 woman with a history of two prior Cesarean deliveries desires a tubal ligation for permanent sterilization. She has two daughters, who are 3 and 1 years old. She is very sure she does not desire any more children. She is happily married and is a stay-at-home-mom. What is the strongest predictor of post-sterilization regret for this patient? A. Not working outside the home B. Parity C. Marital status D. Age E. Children's gender

D. Approximately 10% of women who have been sterilized regret having had the procedure with the strongest predictor of regret being undergoing the procedure at a young age. The percentage expressing regret was 20% for women less than 30 years old at the time of sterilization. For those under age 25, the rate was as high as 40%. The regret rate was also high for women who were not married at the time of their tubal ligation, when tubal ligation was performed less than a year after delivery, and if there was conflict between the woman and her partner.

A 54-year-old G2P2 presents for her health maintenance examination. She has a history of breast cancer treated with mastectomy with reconstruction, chemotherapy, and is currently on tamoxifen. She has been in remission for two years and has been menopausal since the initiation of her chemotherapy. She experiences very mild hot flashes, and is not sleeping well. She appears apprehensive during the examination, although her examination is completely normal except for severe vaginal atrophy. At the conclusion of the office visit, she finally opens up and admits that she has a new boyfriend. She has not had a sexual relationship since her divorce five years earlier, but has been enjoying masturbation. Although excited about initiating sexual activity again, she is obviously concerned. Which of the following is most likely to contribute to sexual dysfunction? A. Sexual desire B. Arousal C. Orgasm D. Dyspareunia E. Body image

D. Female sexual dysfunction can be classified as disorders in sexual desire, arousal, orgasm, or sexual pain, and can include any combination of these. In this case, because she states a desire to initiate in sexual activity and has been enjoying masturbation, it is unlikely that she will experience any problems related to desire, arousal, or orgasm. However, in the presence of severe atrophy and lack of estrogen, she may in fact experience pain related to dyspareunia. She should be encouraged to use some form of water-based lubricant to diminish the effects of the vaginal dryness since estrogen is likely contraindicated with her breast cancer diagnosis. Although body image may play a role, it would be classified under the category of sexual desire.

A 24-year-old woman complains of cyclic mastalgia since the onset of her period at age 12. The symptoms have increased over the years but were less troublesome when she took oral contraceptives a few years ago. Currently, she takes no medications and is not sexually active. She is a strict vegetarian and eats soy products. She does not smoke and she drinks a glass of wine three times a week, and several diet colas every day. Her mother was diagnosed with breast cancer at age 55. Her breast exam is normal, except for some mild fibrocystic changes. Which of the following elements in her history contributes to her increasing pain? A. Alcohol intake B. Vegetarian diet C. Family history of breast cancer D. Caffeine intake E. Age at menarche

D. Fibrocystic breast changes are the most common type of benign breast conditions and occur most often during the reproductive years. Fibrocystic disease is often associated with cyclic mastalgia, possibly related to a pronounced hormonal response. Caffeine intake can increase the pain associated with fibrocystic breast changes, so recommending that she decrease her caffeine intake may be helpful. Pain is not related to alcohol intake, her vegetarian diet or age of menarche. There is an increased risk of breast cancer when atypia is present.q

A 42-year-old G3P3 woman comes to the office after noticing a breast mass while performing a breast self-exam. She is in good health and has normal menstrual cycles. Family history is significant for multiple first and second-degree relatives having breast cancer. Physical exam reveals a 2 cm dominant breast mass. The remainder of the exam is normal. A mammogram obtained today shows no abnormalities. What is the most appropriate next step in the management of this patient? A. Reassurance and observation B. Obtain genetic testing for BRCA1 and BRCA2 C. MRI of the breast D. Fine needle aspiration E. Repeat mammogram in two months

D. Given her positive family hx, can assume that the mammogram might be a false negative. she is >25yo, so go for the FNA.

A 37-year-old G3P3 woman presents for contraceptive counseling. She and her husband have decided that they no longer plan to have children and desire permanent sterilization. Her husband refuses to have a vasectomy. On exam, her BMI is 52; blood pressure is 140/80; and heart rate is 86. She has had three previous Cesarean deliveries. Which of the following options would be the be the best method of permanent sterilization? A. Laparoscopic tubal ligation B. Mini-laparotomy with tubal ligation C. Hysterectomy D. Hysteroscopic tubal occlusion (Essure) E. Endometrial ablation

D. Hysteroscopic tubal occlusion is the best option for this patient. Hysteroscopic tubal occlusion (Essure®) can be performed in the office and places coils into the fallopian tubes that cause scarring that blocks the tubes. Patients are required to use a back up method of contraception for three months following the procedure until a hysterosalpingogram is performed confirming complete occlusion of the tubes. While tubal ligation, either by laparoscopy or mini-laparotomy, are common and effective forms of permanent sterilization, for this patient with her BMI and previous surgeries, this would carry more surgical risks. Hysterectomy is not an indicated procedure for sterilization. Endometrial ablation, or thermal destruction of the endometrial tissue, is an effective treatment for menorrhagia but is not reliable for permanent sterilization.

A 32-year-old G3P2 woman with a last menstrual period two weeks ago presents with a six-month history of abdominal pain. She has noncyclic intermittent pain, which she describes as crampy and diffuse across the lower abdomen. Her pain is typically relieved with defecation and is associated with loose, watery stools. Onset of the symptoms is associated with a change in stool frequency from once daily to multiple times daily. She also experiences bloating and abdominal distention several times a week. Her medical history is significant for chronic migraines and she denies previous surgery. Her gynecological history is unremarkable. Her abdominal examination is notable for mild tenderness to palpation in the left lower quadrant, and her pelvic examination is normal. What is the most likely diagnosis in this patient? A. Pelvic adhesions B. Diverticulosis C. Endometriosis D. Irritable bowel syndrome E. C. difficile colitis

D. Irritable bowel syndrome (IBS) is a common functional bowel disorder of uncertain etiology. It is characterized by a chronic, relapsing pattern of abdominal and pelvic pain, and bowel dysfunction with constipation or diarrhea. IBS is one of the most common disorders associated with chronic pelvic pain. IBS appears to occur more commonly in women with chronic pelvic pain than in the general population. Diagnosis is based on the Rome II Criteria for IBS, which includes at least 12 weeks (need not be consecutive) in the preceding 12 months of abdominal discomfort or pain that has two of three features: 1) relief with defecation; 2) onset associated with a change in frequency of stool; or 3) onset associated with a change in stool form or appearance. The patient's history does not support pelvic adhesions, and diverticulosis (although very common) typically may be asymptomatic unless inflammation/infection develops. In this case, the symptoms for IBS may be indistinguishable from diverticulitis or severe diverticular disease. Although severe endometriosis may affect the lower bowel with constricting and invasive implants, the lack of any gynecologic/menstrual symptoms and the normal pelvic examination essentially excludes this diagnosis. The lack of recent antibiotic exposure essentially rules out the diagnosis of C. difficile.

A 32-year-old G3P1 woman presents to your office today because of exposure to hepatitis B. She had vaginal and anal intercourse with a new partner three days ago and did not use condoms. The partner informed her today he was recently diagnosed with acute hepatitis B acquired from intravenous drug use and needle sharing. She has no prior history of hepatitis B infection and has not been vaccinated. She is currently asymptomatic and her examination is normal. Her urine pregnancy test is negative. What is the next best step in the management of this patient? A. Check AST, ALT, and HBsAg B. Administer HBIG one dose C. Administer HBIG two doses D. Administer HBIG and start hepatitis B vaccine series E. Administer hepatitis B vaccine series only

D. It is estimated that 38% of hepatitis B cases worldwide are acquired from sexual transmission. Post-exposure prophlaxis should be inititated as soon as possible but not later than 7 days after blood contact and within 14 days after sexual exposure. In individuals who are unvaccinated but exposed to persons who are HBsAG positive, recommendations are to receive one dose of HBIG (Hepatitis B Immune Globulin) and the HBV (Hepatitis B Vaccine Series). If the source is HBsAG negative or unknown status, then only the HBV series is used. If the exposed individual has been vaccinated and is a responder then no further treatment is necessary. If the exposed individual is vaccinated and a non-responder, then HBIG plus HBV or HBIG times two doses is used. Because the incubation period for the virus is six weeks to six months, checking liver function and immunologic status at this time is not indicated.`

A 25-year-old G1P0 woman at six weeks gestation comes to the office because of undesired pregnancy. You discuss with her the risks and benefits of surgical versus medical abortion using misoprostol and mifepristone. Compared to surgical abortion, which of the following is increased in a woman undergoing a medical abortion? A. Post abortion pain B. Lower failure rate C. Long-term psychological sequelae D. Blood loss E. Future infertility

D. Medical abortion is associated with higher blood loss than surgical abortion. Early in pregnancy (less than 49 days) both medical and surgical procedures can be offered. Mifepristone (an antiprogestin) can be administered, followed by misoprostol (a prostaglandin) to induce uterine contractions to expel the products of conception. This approach has proven to be effective (96%) and safe. A surgical termination is required in the event of failure or excessive blood loss. Medical termination may be more desirable by some patients since they do not have to undergo a surgical procedure. It does not affect future fertility. Any termination of pregnancy, whether medical or surgical, can have psychological sequelae.

A 19-year-old G0 woman presents to the office with a two-week history of low pelvic pain and cramping. She has a new sexual partner and is on oral contraception and uses condoms. She is one week into her cycle. She has noted no vaginal discharge, itch or odor. She denies fevers or chills. She does note that she is on a new diet and has started drinking lots of water. As such, she notes that she is urinating much more frequently. Her examination is entirely unremarkable. Which of the following is the most appropriate next step in the management of this patient? A. Pelvic ultrasound B. Pap test C. Wet prep D. Urinalysis . Testing for chlamydia

D. Mildly symptomatic or asymptomatic urinary tract infections are common in female patients. Urinary tract infection must be considered in patients who present with low pelvic pain, urinary frequency, urinary urgency, hematuria or new issues with incontinence. While yearly screening for chlamydia is recommended for patients less than 25 years old, this patient's symptoms are most consistent with a UTI. A pelvic ultrasound is not indicated at this point.

A 70-year-old G3P3 woman presents with a four-year history of constant leakage of urine. Her history is significant for abdominal hysterectomy and bilateral salpingo-oophorectomy for endometriosis. She had two anterior repairs in the past for recurrent cystocele. The leakage started six months after her last anterior repair. Pelvic examination reveals no evidence of pelvic relaxation. The vagina is well-estrogenized. Q-tip test reveals a fixed, immobile urethra. Cystometrogram shows no evidence of detrusor instability. Cystourethroscopy showed no evidence of any fistula and reveals a "drain pipe" urethra. Which of the following is the best first treatment for this patient? A. Retropubic urethropexy B. Needle suspension C. Artificial urethral sphincter D. Urethral bulking procedure E. Sling procedure

D. This is a classic example of intrinsic sphincteric deficiency. Urethral bulking procedures are minimally invasive and have a success rate of 80% in these specific patients. The success rates for retropubic urethropexies, needle suspension and slings are less than 50%. An "obstructive or tight" sling can be performed to increase the success rate, but the voiding difficulties are significant, even requiring prolonged or lifelong self-catheterization. Artificial sphincters should be used in patients as a last resort.

A 35-year-old G1P0 woman with last menstrual period one week ago presents with an eight-month history of pelvic pain. She reports regular menstrual cycles with moderate flow and dysmenorrhea, relieved with ibuprofen. She describes her pain as a deep, achy sensation with frequent sharp exacerbations. She has not been sexually active for the last several months because of dyspareunia and some arguments with her new partner of one year. She has no history of sexually transmitted infections. Her medical history is significant for irritable bowel syndrome, managed with a fiber supplement. She is a business executive. She has smoked one pack of cigarettes a day since age 25, and drinks a glass of wine three times a week. She tries to exercise regularly by running three to four times a week. This new pain is distinctly different from her IBS symptoms. Which of the following risk factors can contribute to increased incidence of pelvic pain in this patient? A. Alcohol use B. Smoking habit C. Occupation D. New partner E. Age

D. Most published evidence suggests a significant association of physical and sexual abuse with various chronic pain disorders. The arguments with the new partner allude to possible abuse. Studies have found that 40-50% of women with chronic pelvic pain have a history of abuse. Whether abuse (physical or sexual) specifically causes chronic pelvic pain is not clear, nor is a mechanism established by which abuse might lead to the development of chronic pelvic pain. Women with a history of sexual abuse and high somatization scores have been found to be more likely to have non-somatic pelvic pain, suggesting the link between abuse and chronic pelvic pain may be psychologic or neurologic. However, studies also suggest that trauma or abuse may also result in biophysical changes, by literally heightening a person's physical sensitivity to pain. While smoking can be associated with dysmenorrhea, she has been smoking for 10 years and her symptoms are recent onset.

A 33-year-old G2P2 woman reports a two-year history of severe dysmenorrhea, menorrhagia and pelvic pain following the delivery of her last child. She describes her pelvic pain as primarily in the right lower quadrant, radiating into the vagina. Her pain worsens throughout the day with standing and is associated with pelvic pressure and fullness. Her pelvic examination reveals a mildly enlarged uterus with marked tenderness to palpation of the right adnexa, and no other significant findings. A vaginal ultrasound with color-flow Doppler reveals multiple dilated vessels traversing the right broad ligament to the lower uterus and cervix. The uterus shows no fibroids or other significant changes. Endometrial thickness appears normal. Which of the following is the most likely diagnosis in this patient? A. Endometriosis B. Endometritis C. Adenomyosis D. Pelvic congestion E. Pelvic floor relaxation

D. Pelvic congestion syndrome is a cause of chronic pelvic pain occurring in the setting of pelvic varicosities. The unique characteristics of the pelvic veins make them vulnerable to chronic dilatation with stasis leading to vascular congestion. These veins are thin walled and unsupported, with relatively weak attachments between the supporting connective tissue. The cause of pelvic vein congestion is unknown. Hormonal factors contribute to vasodilatation when pelvic veins are exposed to high concentration of estradiol, which inhibits reflex vasoconstriction of vessels, induces uterine enlargement with selective dilatation of ovarian and uterine veins. This pain may be of variable intensity and duration, is worse premenstrually and during pregnancy, and is aggravated by standing, fatigue and coitus. The pain is often described as a pelvic "fullness" or "heaviness," which may extend to the vulvar area and legs. Associated symptoms include vaginal discharge, backache and urinary frequency. Menstrual cycle defects and dysmenorrhea are common. No signs of pelvic floor relaxation were noted on exam.

A 23-year-old woman presents with complaints of a bilateral nipple itchy sensation for six months. There is no nipple discharge or dry skin. She reports her nipple appears to be swollen at times and there is an erythematous fine rash. She had breast implants placed five years ago, but otherwise has no significant medical problems or surgical history. What is the most likely cause of her symptoms? A. Fibroadenoma B. Breast cancer C. Rupture of breast implants D. Chemical irritants E. Mastitis

D. She's young, likely not to be breast cancer.

A 25-year-old G1P1 woman presents with complaints of a white, watery nipple discharge for four months. She discontinued breastfeeding six months ago. She has been told in the past she had fibrocystic breast changes, but otherwise has no significant medical problems or surgical history. A white nipple discharge is noted on manual expression, but the exam is otherwise normal. She was then sent for a serum prolactin level which was 45 ng/ml (normal below 40 ng/ml). What is the most appropriate next step in the management of this patient? A. Obtain a brain MRI B. Obtain a beta-hCG C. Begin Bromocriptine D. Obtain a fasting prolactin level E. Order a ductogram

D. Stimulation of the breast during the physical examination may give rise to an elevated prolactin level. Accurate prolactin levels are best obtained in the fasting state. If still elevated, then a TSH level and brain MRI would be indicated to rule out a pituitary tumor. Post partum women may continue to produce milk for up to two years after cessation of breastfeeding. Although pathologic factors such as hypothyroidism, hypothalamic disorders, pituitary disorders (adenomas, empty sella syndrome), chest lesions (breast implants, thoracotomy scars, and herpes zoster) and renal failure can elevate prolactin levels, a non-significant benign elevation needs to be ruled out first. A ductogram is usually indicated in patients who have bloody discharge from a single breast duct.

A 23-year-old G1 woman with six weeks amenorrhea presents with lower abdominal pain and vaginal bleeding. Her temperature is 102.0°F (38.9°C) and the cervix is 1 cm dilated. Uterus is eight-week size, tender and there are no adnexal masses. Urine pregnancy test is positive. Which of the following is the most appropriate next step in the management of this patient? A. Observation B. Single-agent antibiotics C. Medical termination of pregnancy plus antibiotics D. Uterine evacuation plus antibiotics E. Laparoscopy plus antibiotics

D. The management of septic abortion includes broad-spectrum antibiotics and uterine evacuation. Single agent antimicrobials do not provide adequate coverage for the array of organisms that may be involved and therefore are not indicated. A laparoscopy can be indicated if ectopic pregnancy is suspected, but it is unlikely in this case. Medical termination is not the best option since prompt evacuation of the uterus is indicated for septic abortion.

A 23-year-old G2P1 woman with six weeks amenorrhea presents with lower abdominal pain and vaginal bleeding. Her temperature is 102.0°F (38.9°C) and the cervix is 1 cm dilated. Uterus is eight-week size and tender. There are no adnexal masses. Urine pregnancy test is positive. What is the most likely diagnosis? A. Threatened abortion B. Missed abortion C. Normal pregnancy D. Septic abortion E. Ectopic Pregnancy

D. The patient has a septic abortion. She has fever and bleeding with a dilated cervix which are findings seen with septic abortion. Threatened abortions clinically have vaginal bleeding, a positive pregnancy test and a cervical os closed or uneffaced. Missed abortions have retention of a nonviable intrauterine pregnancy for an extended period of time (i.e. dead embryo or blighted ovum). A normal pregnancy would have a closed cervix. Ectopic pregnancy would likely present with bleeding, abdominal pain, possibly have an adnexal mass, and the cervix would typically be closed.

A 26-year-old G0 women returns for a follow-up visit regarding suspected endometriosis. She has been using NSAIDs and oral contraceptive pills to help manage her pelvic pain which has been getting worse. While discussing further treatment options, she asks if there is any test or procedure you can perform to confirm her diagnosis. Which of the following would you recommend? A. CA-125 B. Pelvic ultrasound C. CT scan of the abdomen and pelvis D. MRI of the pelvis E. Diagnostic laparoscopy

E. Definitive diagnosis is based on exploratory surgery and biopsies, although endometriosis is usually initially treated based on the clinical presentation. In addition, this patient can benefit from laparoscopy, since she has failed the two most common treatments for endometriosis, NSAIDs and OCPs. There is no imaging study or blood test that can confirm the diagnosis of endometriosis. CA-125 is non-specific and can be elevated in patients with endometriosis, and therefore not helpful.

A 32-year-old G0 woman comes to your office because she has been unable to conceive for one year. She is currently in a mutually monogamous relationship with her husband, has intercourse three times per week, and has no dyspareunia. Her menstrual cycles occur every 26-34 days. She has had seven sexual partners in the past. She was treated for multiple sexually transmitted infections including gonorrhea, chlamydia and pelvic inflammatory disease in her early twenties. She had an abnormal Pap test about four years ago and was treated with a LEEP. What is the most likely underlying cause of infertility in this patient? A. Luteal phase defect B. Cervical stenosis C. Ovulatory dysfunction D. Tubal disease E. Endometriosis

D. The rate of tubal infertility has been reported as 12% after one episode of PID, 25% after 2 episodes and 50% after three episodes. Salpingitis can develop in 15-30% of women with inadequately treated gonococcal or chlamydial infections and can result in significant long-term sequelae, such as chronic pelvic pain, hydrosalpinx, tubal scarring and ectopic pregnancy. Given this patient's history, her inability to conceive is most likely due to the long-term sequelae of a sexually transmitted infection. Although the patient had a LEEP, risk for cervical stenosis is low. She is having regular cycles; therefore, anovulation and luteal phase defect is less likely. This case emphasizes the importance of aggressive screening and treatment protocols for sexually transmitted infections, as well as counseling regarding abstinence and safer sex practices. While endometriosis can cause tubal occlusion, her clinical presentation is not consistent with endometriosis.

A 26-year-old G0 woman presents with severe right lower quadrant pain associated with nausea for the last six hours, which began shortly after she finished her aerobic exercises. She has a history of suspected endometriosis, which was diagnosed two years ago, based on her severe dysmenorrhea. She has been using NSAIDs during her menses to control the pain. She is not sexually active, and is otherwise in good health. Her menstrual cycles are regular and her last menstrual period was three weeks ago. She has no history of sexually transmitted infections. Her vital signs are: blood pressure 145/70; pulse 100; temperature 99.2°F (37.3°C). She appears uncomfortable. On abdominal examination, she has moderate tenderness to palpation in the right lower quadrant. On pelvic exam, she has no lesions or discharge. A thorough bimanual examination is difficult to perform due to her discomfort. Beta-hCG <5 mIU/ml and WBC 8,500 /microliter. A pelvic ultrasound shows a 6 cm right ovarian mass. The uterus and left ovary appear normal. There is a moderate amount of free fluid in the pelvis. What is the most likely diagnosis in this patient? A. Appendicitis B. Exacerbation of the endometriosis C. Ovarian carcinoma D. Ovarian torsion E. Ectopic pregnancy

D. The sudden onset of pain and nausea, as well as the presence of a cyst on ultrasound, suggest ovarian torsion. Although appendicitis is in the differential diagnosis, it is unlikely to have such a sudden onset of pain and a normal white count. Her endometriosis can get worse but it would be unlikely to be of such sudden onset. Although she has an adnexal mass, a negative Beta-hCG rules out pregnancy.

A 23-year-old G1P0 woman at 10 weeks gestation presents with an intrauterine embryonic demise. On exam, her blood pressure is 120/80; heart rate is 67; and she is afebrile. Her cervix is closed and there is no evidence of bleeding. She desires to have the most minimally invasive treatment as possible. Which of the following options is best for this patient? A. Dilation and curettage B. Manual vacuum aspiration C. Mifepristone D. Misoprostol E. Oxytocin

D. This patient has a missed abortion. Expectant management is the least invasive treatment. Of the options listed, Misoprostol (prostaglandin E1) is the least invasive best option for this patient. Misoprostol can be administered orally or vaginally and will induce uterine cramping with expulsion of products of conception. Potential risk factors of use include hemorrhage as well as failure. Dilation and curettage and manual vacuum aspiration are effective methods for treatment of a missed abortion, but are invasive procedures. Mifepristone, a progesterone receptor blocker, is used for pregnancy termination. It is recommended for use within 49 days of the last menstrual period, but there is data to show that it can be effective up to nine weeks. Oxytocin would not be effective as there has not been up-regulation of receptors at this gestational age.

A 56-year-old G3P3 woman presents to the office for her annual health maintenance examination. She is in good health and is not taking any medications. She has been postmenopausal for three years. She had an abnormal Pap test 10 years ago, but results have been normal every year since. She is sexually active with her husband. On examination, her cervix is 1 cm above the vaginal introitus and there is moderate bladder prolapse. Her uterus is normal in size and she has no adnexal masses or tenderness. In addition to recommending a mammogram, what is the most appropriate next step in the management of this patient? A. Cystocele repair B. Pelvic ultrasound C. Total hysterectomy D. Observation E. Topical estrogen

D. This patient is asymptomatic from her prolapse; therefore, no intervention is necessary at this point. Cystocele repairs and hysterectomies are invasive procedures which are not indicated in this asymptomatic patient. It is not necessary to obtain a pelvic ultrasound, as her uterus is normal in size and she has no adnexal masses. Topical estrogen would not help improve the prolapse, although it might help with her vaginal dryness. She seems to be doing well with the lubricants and it is not necessary to expose her to estrogen.

A 26-year-old G2P2 woman presents with urinary urgency and dysuria for the past three days. She has a history of a urinary tract infection once. She is sexually active and uses condoms for contraception. She is otherwise healthy and does not take any medications or supplements. She does not have fever, chills, flank pain or vaginal discharge. Which of the following organisms is the most likely cause of this patient's symptoms? A. Enterococcus faecalis B. Klebsiella pneumoniae C. Proteus mirabilis D. Staphylococcus saprophyticus E. Escherichia coli

E. Acute cystitis in a healthy, non-pregnant woman is considered uncomplicated and is very common. Escherichia coli causes 80 to 85 percent of cases. The other major pathogens are Staphylococcus saprophyticus, Klebsiella pneumoniae, Enterococcus faecalis and Proteus mirabilis. The physician must consider antibiotic resistance when determining treatment.

A 29-year-old G3P0 woman presents for evaluation and treatment of pregnancy loss. Her past medical history is remarkable for three early (<14 weeks gestation) pregnancy losses. Parental karyotype was normal. Which of the following is the most appropriate next step in the management of this patient? A. Prophylactic cerclage with her next pregnancy B. Serial cervical length with her next pregnancy C. 17-hydroxyprogesterone with her next pregnancy D. Check for Factor V Leiden mutation E. Check antiphospholipid antibodies

E. Antiphosphospholipid antibodies are associated with recurrent pregnancy loss. The workup for antiphospholipid syndrome includes assessment of anticardiolipin and beta-2 glycoprotein antibody status, PTT, and Russell viper venom time. Recurrent pregnancy loss is defined as > two consecutive or > three spontaneous losses before 20 weeks gestation. Etiologies include anatomic causes, endocrine abnormalities such as hyper- or hypothyroidism and luteal phase deficiency, parental chromosomal anomalies, immune factors such as lupus anticoagulant and idiopathic factors. Her history is not consistent with cervical insufficiency which is diagnosed in the second trimester by history, physical exam and other diagnostic tests, such as ultrasound. Serial cervical lengths or placement of a cerclage are not indicated in this patient. Treatment with 17-hydroxyprogesterone is indicated in patients with a history of prior preterm birth. Factor V Leiden mutation has not been associated with recurrent pregnancy loss. It can be associated with thrombotic events.

A 32-year-old G3P3 woman comes to the office to discuss permanent sterilization. She has a history of hypertension and asthma (on corticosteroids). She has been married for 10 years. Vital signs show: blood pressure 140/90; weight 280 pounds; height 5 feet 9 inches; and BMI 41.4kg/m2. You discuss with her risks and benefits of contraception. Which of the following would be the best form of permanent sterilization to recommend for this patient? A. Laparoscopic bilateral tubal ligation B. Mini laparotomy tubal ligation C. Exploratory laparotomy with bilateral salpingectomy D. Total abdominal hysterectomy E. Vasectomy for her husband

E. Both vasectomy and tubal ligation are 99.8% effective. Vasectomies are performed as an outpatient procedure under local anesthesia, while tubal ligations are typically performed in the operating room under regional or general anesthesia; therefore carrying slightly more risk to the woman, assuming both are healthy. She is morbidly obese, so the risk of anesthesia and surgery are increased. In addition, she has chronic medical problems that put her at increased risk of having complications from surgery.

A 23-year-old G0 woman with last menstrual period 14 days ago presents to the office because she had unprotected intercourse the night before. She does not desire pregnancy at this time and is requesting contraception. She has no medical problems and is not taking any medications. In addition to offering her counseling and testing for sexually transmitted infections, which of the following is the most appropriate next step in the management of this patient? A. Observation for two weeks to establish if pregnancy occurred before initiating treatment B. Oral contraceptives now C. Oral contraceptives after her next normal menstrual period D. Emergency contraception and follow-up after next menstrual period E. Provide emergency contraception, then begin oral contraceptives immediately

E. Emergency contraceptive pills are not an abortifacient, and they have not been shown to cause any teratogenic effect if inadvertently administered during pregnancy. They are more effective the sooner they are taken after unprotected intercourse, and it is recommended that they be started within 72 hours, and no later than 120 hours. Plan B®, the levonorgestrel pills, can be taken in one or two doses and cause few side effects. Emergency contraceptive pills may be used anytime during a woman's cycle, but may impact the next cycle, which can be earlier or later with bleeding ranging from light, to normal, to heavy.

A 35-year-old G3P3 woman requests contraception. Her youngest child is seven years old. Her periods have been regular since she discontinued breastfeeding five years ago. Her past medical history includes depression that is controlled with antidepressants, and a history of deep venous thrombosis. She denies smoking or alcohol use. In the past, oral contraceptive pills have caused her to have severe gastrointestinal upset. What in her history makes her an ideal candidate for progestin-only pills? A. Depression B. Smoking history C. Severe nausea on combined oral contraceptives D. Lactation history E. Deep venous thrombosis

E. Ideal candidates for progestin-only pills include women who have contraindications to using combined oral contraceptives (estrogen and progestin containing). Contraindications to estrogen include a history of thromboembolic disease, women who are lactating, women over age 35 who smoke or women who develop severe nausea with combined oral contraceptive pills. Progestins should be used with caution in women with a history of depression.

A 24-year-old G1P1 woman comes to the office requesting contraception. Her past medical history is unremarkable, except for a family history of ovarian cancer. She denies alcohol, smoking and recreational drug use. She is in a monogamous relationship. She wants to decrease her risk of gynecological cancer. Of the following, what is the best method of contraception for this patient? A. Diaphragms B. Condoms C. Copper containing intrauterine device D. Progesterone containing intrauterine device E. Combined oral contraceptives

E. Oral contraceptives will decrease a woman's risk of developing ovarian and endometrial cancer. The first developed higher dose oral contraceptive pills have been linked to a slight increase in breast cancer, but not the most recent (current) lower dose pills. Women who use oral contraceptive pills have a slightly higher risk of developing cervical intraepithelial neoplasia, but their risk of developing PID, endometriosis, benign breast changes and ectopic pregnancy are reduced. Both hypertension and thromboembolic disorders can be a potential side effect from using oral contraceptive pills. Diaphragms, condoms and intrauterine devices will not lower her risk of ovarian cancer. The progesterone IUD may decrease a woman's risk for endometrial cancer but would not effect her risk for ovarian cancer, and have been associated with increased ovarian cysts.

A 54-year-old G2P2 presents for a health maintenance examination. She has a history of breast cancer treated with mastectomy with reconstruction, chemotherapy, and is currently on tamoxifen. She has been in remission for two years and has been menopausal since the initiation of her chemotherapy. She experiences very mild hot flashes and is not sleeping well. She appears apprehensive during the examination, although her examination is completely normal except for some mild vaginal atrophy. At the conclusion of the office visit, she finally opens up and admits that she has a new boyfriend after having gone through a divorce five years earlier. She is anxious about initiating sexual activity again and wants your advice on what she should do. Which of the following is likely contributing to her anxiety? A. Breast cancer diagnosis B. Menopausal symptoms C. Body image D. Vaginal atrophy E. All of the above

E. Taking a good sexual history is critical to understanding the root cause of a woman's concerns about possible sexual dysfunction. Etiologies may be physiologic, hormonal, psychologic, and often multifactorial. To distinguish among these causes, a thorough inquiry into a woman's hormonal, sexual, medical, and social history is necessary. A sexual history must include a non-judgemental and candid discussion related to her interests, desires, and practices to better understand potential causes for concern and dysfunction.

A 35-year-old G3P3 woman comes to the office because she desires contraception. Her past medical history is significant for Wilson's disease, chronic hypertension and anemia secondary to menorrhagia. She is currently on no medications. Her vital signs reveal a blood pressure of 144/96. Which of the following contraceptives is the best option for this patient? A. Progestin-only pill B. Low dose combination contraceptive C. Continuous oral contraceptive D. Copper containing intrauterine device E. Levonorgestrel intrauterine device

E. The levonorgestrel intrauterine device has lower failure rates within the first year of use than does the copper containing intrauterine device. It causes more disruption in menstrual bleeding, especially during the first few months of use, although the overall volume of bleeding is decreased long-term and many women become amenorrheic. The levonorgestrel intrauterine device is protective against endometrial cancer due to release of progestin in the endometrial cavity. She is not a candidate for oral contraceptive pills because of her poorly controlled chronic hypertension. The progestin-only pills have a much higher failure rate than the progesterone intrauterine device. She is not a candidate for the copper-containing intrauterine device because of her history of Wilson's disease.

A 17-year-old G0 sexually active female presents to the emergency department with pelvic pain that began 24 hours ago. She reports menarche at the age of 15 and coitarche soon thereafter. She has had four male partners, including her new boyfriend of a few weeks. Her blood pressure is 100/60; pulse 100; and temperature 102.0°F (38.9°C). On speculum examination, you note a foul-smelling mucopurulent discharge from her cervical os and she has significant tenderness with manipulation of her uterus. What is the next best step in the management of this patient? A. Outpatient treatment with oral broad spectrum antibiotics B. Outpatient treatment with intramuscular and oral broad spectrum antibiotics C. Intravenous antibiotics and dilation and curettage D. Inpatient treatment, laparoscopy with pelvic lavage E. Inpatient treatment and intravenous antibiotics

E. The most likely cause of the symptoms and signs in this patient is infection with a sexually transmitted organism. The most likely organisms are both N. gonorrhoeae and chlamydia, and the patient should be treated empirically for both after appropriate blood and cervical cultures are obtained. There is no evidence that adolescents have better outcomes from inpatient therapy. However, since the patient also has a high fever, inpatient admission is recommended for aggressive intravenous antibiotic therapy in an effort to prevent scarring of her fallopian tubes and possible future infertility.

A 16-year-old G0 female presents to the emergency department with a two-day history of abdominal pain. She is sexually active with a new partner and is not using any form of contraception. Temperature is 101.8°F (38.8°C). On examination, she has lower abdominal tenderness and guarding. On pelvic exam, she has diffuse tenderness over the uterus and bilateral adnexal tenderness. Beta-hCG is <5. What is the most likely diagnosis for this patient? A. Ectopic pregnancy B. Appendicitis C. Acute cystitis D. Endometriosis E. Acute salpingitis

E. The signs and symptoms of acute salpingitis can vary and be very subtle with mild pain and tenderness, or the patient can present in much more dramatic fashion with high fever, mucopurulent cervical discharge and severe pain. Important diagnostic criteria include lower abdominal tenderness, uterine/adnexal tenderness and mucopurulent cervicitis.

A 23-year-old G1P0 woman at six weeks gestation undergoes a medical termination of pregnancy. One day later, she presents to the emergency department with bleeding and soaking more than a pad per hour for the last five hours. Her blood pressure on arrival is 110/60; heart rate 86. On exam, her cervix is 1 cm dilated with active bleeding. Hematocrit on arrival is 29%. Which of the following is the most appropriate next step in the management of this patient? A. Admit for observation B. Repeat hematocrit in six hours C. Begin transfusion with O-negative blood D. Give an additional dose of prostaglandins E. Perform a dilation and curettage

E. This patient is having heavy bleeding as a complication of medical termination of pregnancy. The most likely etiology for her bleeding is retained products of conception. This is managed best by performing a dilation and curettage. It is not appropriate to wait six hours before making a decision regarding next step in management, or to just admit her for observation. Since the patient is not symptomatic from her anemia, it is not necessary to transfuse her at this time.

A 26-year-old G0 presents to the emergency department with eight hours of severe right lower quadrant pain associated with nausea. She has a history of suspected endometriosis, which was diagnosed two years ago based on severe dysmenorrhea. She has been using NSAIDs during her menses to control the pain. She is not sexually active. She is otherwise in good health. Her menstrual cycles are regular and her last menstrual period was three weeks ago. She has no history of sexually transmitted infections. Her vital signs are: blood pressure 145/70; pulse 100; temperature 98.6°F (37.0°C). She appears uncomfortable. On abdominal examination, she has moderate tenderness to palpation in the right lower quadrant. On pelvic examination, she has no lesions or discharge. A thorough bimanual examination was difficult to perform due to her discomfort. Beta-hCG <5 mIU/ml and hematocrit 29%. A pelvic ultrasound shows a 6 cm right ovarian mass. The uterus and left ovary appear normal. There is a moderate amount of free fluid in the pelvis. What is the most appropriate next step in the management of this patient? A. Begin oral contraceptives B. MRI of the pelvis C. Doppler pelvic ultrasound D. CT scan of the pelvis E. Surgical exploration

E. This patient most likely has ovarian torsion and needs to be surgically explored. Further imaging studies will not help beyond the information obtained on the ultrasound. A Doppler ultrasound to check the blood flow to the ovaries is controversial, as normal flow does not rule out ovarian torsion. Although oral contraceptives can help decrease the development of further cyst formation and control the pain associated with endometriosis, this patient needs immediate surgical attention due to suspected ovarian torsion.


Kaugnay na mga set ng pag-aaral

CS271 - Module 1 Notes & Knowledge Checks

View Set

Special Positioning- Upper Extremities

View Set

Chapter 10B Offensive and Defensive Strategies

View Set